You are on page 1of 36

Countdown Mock 15 (CLAT) 2023

English Language
Directions: Read the passages given below and answer the questions that follow.

Passage - 1

In the present age, the theory of legal equality is given recognition in a natural way but it has taken to get this
recognition. In
the ancient societies the idea of legal equality was almost unknown. For example under Manusmriti
for the same crime, there
is a provision of different punishments for different castes. [1] This meant the lower
caste of the criminal, the harder
punishment will be given. Similarly Aristotle has given the suggestion that for "the
same crime, a slave should get more
severe` punishment than a freeman because slave is less sensitive." In the
history of Europe legal inequality had been very
dominating since 18th Century. Before the French revolution
(1789), elite person could present the evidence from his side in
the court whereas the common had no right to
present evidence in his favour. In England slave had no legal status which
means slave had no personality from
legal point of view. [2] Since 1918 there was a rule that those who would get the poor
relief will have very ordinarily
legal position because they will not have right to vote like others. Till 1870, married women did
not have the legal
right for the ownership of property and till 1918, all the women were deprived of the voting right. These
inequalities
were removed after a long battle.

[3] An expression of legal equality is the equality before law. J.R. Lucus under "Principles of Politics" (1976) has
given a very
appropriate definition of "Equality before Law"; "Equality before law does not make sure that law will
treat everyone equally,
rather it finalises that the door of law will be open for everyone and in any matter only those
things will be thought about which
are relevant under law. No body is so small that he cannot go to the court and
nobody is so powerful that he is not responsible
towards courts. Anybody can demand help from the courts and
everyone is bound to abide by their order. Courts will give
decision only after listening to both parties and without
any fear or partiality and in a right and impartial manner." So the legal
equality means that all the citizen will have
equal subjection to the law and all the citizens will have equal protection of law.

[4] Legal equality is no doubt the foundation stone of legal justice. But in a society where there is a vast gap
between the
wealth and poverty, legal equality is not enough to provide true justice. All people can get equal benefit
from legal equality
only when they are equally capable of taking help of the courts to protect their rights or get
compensation of their loss. Till the
legal expenses remain very high, legal process will remain very complete and
there will be strong economic inequalities in the
country; the possibility of getting equal benefit of the guardianship
of the law will remains very dim. We are well acquaintant
with such situation in India.
Q 1.   What is the author trying to convey through the examples of Manusmriti and Aristotle in the first paragraph?

a)  That both Aristotle and Manusmriti were conservative in their time.

b)  That theory of legal equality is a modern theory like Artificial Intelligence.


c)  That the idea of legal equality in their times was almost unknown.

d)  That a lot of effort has gone into recognising legal equality as a right.

Q 2.   Out of the following, what can be inferred from the information given in the passage?

a)  The notion of legal equality in "Principles of Politics" is blurred.

b)  Legal equality is not only about equal protection of laws.

c)  Courts have played a significant role in recognising the idea of equality.


d)  The contribution of Manusmriti to the legal system should be acknowledged.
Q 3.   What is the tone of J.R. Lucus in his expression of legal equality in "Principles of Politics"?

a)  Coarse b)  Informative c)  Humorous d)  Authoritative

Q 4.   Out of the following statements, which statement contains a grammatical error?

a)  [4] b)  [3] c)  [1] d)  [2]

Q 5.   Which among the following best represents the opinion of the author on the definition of legal equality by
Lucus?

a)  Apt and Congruous b)  Praiseworthy yet faulty c)  Faulty and deceptive d)  Information insufficient

Directions: Read the passages given below and answer the questions that follow.

Passage - 2

The parallels between the warnings from biologists about the loss of biodiversity and from anthropologists about
the loss of
cultural diversity are striking. In both cases, modernity - or globalisation if you wish - is the key cause of
loss. This is a dual
track worth pursuing because the main causes of loss in both cases are the same. Additionally,
loss of diversity, whether
cultural or biological, can be understood as a single phenomenon: the world is becoming
flatter, with less complexity and
fewer options.

These issues are not new. In Roger M Keesing's textbook Cultural Anthropology: A Contemporary Perspective
(1976), taught
to undergraduates in the 1980s, one of the final chapters is titled 'Response to Cataclysm'. It deals
with the effects of state
interventions and capitalist expansion on small-scale societies. Keesing was far from the
first to raise the alarm. As early as
1839, James Cowles Prichard gave an address to the British Association for the
Advancement of Science, where he spoke of
the recent extension of 'the progress of colonisation' and its detrimental
effects on local cultures. He concluded:

A great number of curious problems in physiology, illustrative of the history of the species, and the laws of their
propagation,
remain as yet imperfectly solved. The psychology of native races has been but little studied in an
enlightened manner; and yet
this is wanting in order to complete the history of human nature, and the philosophy
of the human mind. How can this be
obtained when so many tribes shall have become extinct, and their thoughts
shall have perished with them?

As to biological diversity, similar concerns inspired intellectuals and explorers to campaign for the establishment
of national
parks in the 19th century. Seeing nature not as an adversary to be overcome but as a treasure to be
cherished, a broad
palette of engaged citizens - in the United States, they included George Catlin, Henry David
Thoreau, John Muir and Abraham
Lincoln himself - saw unspoilt nature as inherently valuable, and in need of active
protection.

Since the incipient environmentalist movement and the parallel concern among early anthropologists to document
'vanishing
cultures', the visibility of human footprints and global cultural homogenisation have accelerated
dramatically. Both as regards
biological and cultural diversity, the threats are now massive and ominous.
The main causes are the expansion of state and
market forces, and the outcome can be described, in both cases,
as a loss of flexibility. Whenever an insect species vanishes,
or a language loses its last native speaker,
the biosphere loses options.

A useful framework for understanding these related processes towards simplification and homogenisation - that
is, towards
the loss of options - is offered in the emerging field of biosemiotics, which views the biosphere - human
as well as nonhuman -
as a semiosphere: a system of communication defined by the ongoing, continuous exchange
of signs. As its name suggests,
biosemiotics offers a way of interpreting and studying nature, culture and their
mutual entanglements by reading the way
organisms influence each other through a continuous process of
communication.
Q 6.   Out of the following, which option best represents the central idea of the passage?

a)  There is much more in common between biodiversity and cultural diversity than perceived.

b)  The loss of biodiversity and cultural diversity may have similar causes.
c)  Loss of biodiversity is directly related to the loss of cultural diversity and vice versa.

d)  It is more important to preserve biodiversity first than to preserve cultural diversity.

Q 7.   What could be inferred from the information given in the passage?


i. The author does not desire a loss of flexibility in biodiversity or cultural diversity.

ii. Biosemiotics helps develop alternatives when there is a loss of flexibility in either biodiversity or cultural
diversity.

iii. Evidence suggests environmental movements have been carried out since the very inception of humanity.

a)  Only i b)  Only i and ii c)  Only i and iii d)  Only ii and iii

Q 8.   Out of the following, what is a synonym for the word 'incipient'?

a)  Dawning b)  Extraction c)  Middle Stage d)  Neoteric

Q 9.   Which of the following is valid from the speech of James Prichard?

a)  Studying the psychology of species on the verge of extinction is impossible.

b)  The history of human nature is incomplete without adequately studying native species' psychology.

c)  People have used information from this speech to curate their hypotheses.
d)  All of the above

Q 10.   What has been conveyed by the author in the penultimate paragraph of the passage?

a)  The impact of humans on biodiversity was identified at a very early stage.

b)  The biosphere wants more options from the environment, not lesser options.

c)  The impact of humans on cultural diversity has increased over time.


d)  Biological and cultural diversity in the current times faces quite different challenges.

Directions: Read the passages given below and answer the questions that follow.

Passage - 3

Cries for the end of slavery began appearing in American newspapers as early as 1817 and were especially
pronounced in
those of Benjamin Lundy and William Lloyd Garrison, two editors who advocated tirelessly for
abolition. Lundy began
publishing the Genius of Universal Emancipation in 1821, now recognized as one of the
most influential early abolitionist
papers. [1] He traveled most northern states, carrying his newspaper equipment
on his back, lecturing about the evils of
slavery, and producing an issue of his newspaper whenever local printers
would share their facilities. He would then mail the
completed issue to subscribers. Eventually, he found that he
needed a home office for regular publication and hired William
Lloyd Garrison to be his editor. Although both were
ardent supporters of abolition, they differed in their opinions on how to
achieve this goal and eventually parted
ways.

Garrison started his own abolition paper, the Liberator, which first appeared January 31, 1831, in Boston. He was
a radical,
supporting immediate emancipation for all slaves and their enfranchisement, positions which earned
him numerous enemies.
[2] In 1835, a Boston mob attacked Garrison and he was jailed for his himself protection,
but despite threats to his paper and
his life, Garrison continued to print the Liberator for 35 years, making it one of
the most influential newspapers of its time.

The earliest black newspapers also sought to bring about the end of slavery. Two of the most influential black
editors were
Samuel Cornish and John Russwurm, who together in 1827 began publishing Freedom's Journal.
Cornish was a Presbyterian
minister and Russwurm was one of the first blacks to earn a college degree in
America. [3] Together the two committed to
publish a newspaper that would give blacks a voice in the debate over
slavery. Unfortunately, like Lundy and Garrison,
Cornish and Russwurm disagreed over how to bring about the end
of slave holding and in 1829 parted ways. Cornish
renamed the paper Rights of All and continued to publish it until
1830 when financial troubles caused its closure.

Perhaps the most influential black editor was Frederick Douglass, a runaway slave who started his work as a
journalist with
Garrison and later published his own newspaper. Douglass escaped slavery in 1838, traveling to
relative safety in
Massachusetts where he began to lecture about his experiences as a slave and to publish
accounts of his experiences in
Garrison's Liberator. [4] In 1845, afraid his fame would lead to his recapture and
return to slavery, Douglass flown to Great
Britain where he spent two years traveling and speaking until supporters
raised enough money to buy his freedom. Upon his
return to the United States as a free black, Douglass began
publishing the North Star on November 1, 1847. Four years later
he merged his paper with another under the new
title of Frederick Douglass' Paper, which continued publication until 1860
and the eve of the Civil War.
Q 11.   Out of the following, what could be inferred from the information given in this passage?

a)  Lundy and Garrison supported the same pathway even though there goals differed.

b)  The goals of the earliest American newspapers and the black newspapers were different.
c)  The reason for the partition between Lundy and Garrison, and Cornish and John are similar.

d)  All of the above

Q 12.   Choose the option that is consistent with the information presented about black newspapers and editors in
the
passage.

a)  Samuel Cornish had a better way to execute his plans than John Russwurm.

b)  Lundy and Garrison would not have parted if not for their disagreements.

c)  The people of the 1830s decade could buy freedom.

d)  Even though Douglass was the most influential newspaper, his work was not commended.

Q 13.   Which of the incidents given in the passage contain an irony?

a)  The partition between Cornish and John b)  The imprisonment of Garrison

c)  The partition between Garrison and Lundy d)  The inception of Genius of Universal Emancipation

Q 14.   Among the given four statements, which statement does not contain a grammatical error?

a)  [1] b)  [2] c)  [3] d)  [4]

Q 15.   Why was Frederick Douglass captured and put into prison? Choose the most logical option.

a)  Because of his increasing fame among the people.

b)  Because his supporters found out that he was an drug addict.


c)  Because of the wealth that he had accumulated to buy his freedom.

d)  None of the above

Directions: Read the passages given below and answer the questions that follow.

Passage - 4

In cities, the frenetic energy of crowds is feared and fetishised. Fear takes hold when we hear the great din of
stampeding
masses running from a shooting, a fire or a collapsing building. Municipal governments and police
departments feel it too,
when they're required to manage crowds during demonstrations, concerts, sports events
- even public holidays. During these
moments, that fear can lead to violence as police attempt to regain control
using tear gas, truncheons or other methods. At
the same time, we fetishise bustling cities and their crowds.
As cities have grown safer and more luxurious, people are
attracted to the busiest parts of dense urban centres.
We might be drawn to the pleasant buzzing of lunchtime masses moving
through a city like New York or drawn to
the romance of Shenzhen's glittering blanket of electric lights that stretches out to the
horizon. The proximity -
even comfort - of anonymous bodies moving together through a streetscape brings tourists to Tokyo's
Shibuya
Crossing and Istanbul's Taksim Square.

Rubbing shoulders with strangers is considered both a pleasure and a pain of urban life. Density can be an
endless source of
social possibility, of chance encounters in city streets. In Donna Tartt's novel The Goldfinch
(2013), the cities of New York and
Amsterdam are essential to creating an aura of destiny that propels her
Bildungsroman forward. Density can also enforce a
sense of psychological distance and anomie. In Wong
Kar-Wai's film Chungking Express (1994), lovesick characters drift
emotionally alone through the packed, neon-lit
streets of Hong Kong. Though equally feared and fetishised, in the late 20th
and early 21st centuries, crowds and
urban density increasingly became a necessary and even desirable part of life.
However, that shift ended abruptly
in 2020 when the COVID-19 pandemic turned teeming cities into sites of extreme viral risk,
leading some to ask:
is the golden age of urban density finally over?

The pandemic fundamentally challenged our acceptance of crowds and urban density. As the virus spread, residents
with
means left the city and retreated to the countryside. Those who remained developed anxieties about sharing
lifts, public
transport or the communal air vents in apartment buildings. This created a sense of density dread.
Yet, despite this
rebalancing of the scales from fetishisation to fear, the long-term damage to the dream of living in
large, crowded, even
exciting cities seems to have been temporary. Residential property prices are at an all-time
high in many countries, including
inner-city flats, despite warnings of a coming migration to the suburbs in the
wake of the coronavirus pandemic. As housing
prices soar, there have been new demands to increase affordability
by building more apartments. The question of density has
become pressing not just in the United States, where a
home and yard are interpreted as quintessential aspects of the
American Dream, but across the world.

Social scientists have also long struggled with the question of density. To them, the city has often appeared in
contrary terms:
as an engine for immigrant assimilation and economic empowerment; as a site of deviance;
and as a space for cultural
freedom where one can sever constricting familial and social ties. In much of the
world's poorer nations, living in the sprawl
remains a mark of affluence, and density is something to be transcended.
What does an alternative look like? Is another kind
of density possible, with close-knit communities, sustainable
places where car use is minimised, where consumption is
decreased, and where people can walk out their front
door into a unique and enlivening environment?
Q 16.   What is the best representation of the main idea of the third paragraph?

a)  People still desire to live in the countryside even though they envy the hustle and bustle of a crowded city.

b)  Every one in America lives towards fulfilling the American dream of having a house and a yard.

c)  The long term damage to the dream of living in crowded cities seems to have been temporary.

d)  
Despite predictions of a future movement to the cities following the epidemic, it appears that a long-term
damage was done.

Q 17.   Which of the following words is / are synonym to the word 'frenetic'?

a)  Frantic b)  Frenzied c)  Intense d)  All of the above

Q 18.   According to the author, what brings people closer to cities like Tokyo?

a)  The closeness of so many random people that cities like Tokyo provide.
b)  The dins and the noises that cities like Tokyo make during its peak.

c)  The safety of cities like Tokyo give people a lot of assurance.

d)  Cities like Tokyo are known for their efficient public transport systems.

Q 19.   Assuming the information given in the passage to be true, choose the option that can be inferred.

a)  The question of density has long confounded the social scientists in some way.

b)  In the lower income countries, affluent people live in the densest part of cities.
c)  The pandemic played a role in the transition to fear of density from its fetishisation.
d)  Both a and c are correct

Q 20.   What is the purpose of the author in giving the example of Chungking Express?

a)  To demonstrate how the fetishisation of density turned to its fear.

b)  To explain how it is a necessity these days to live in dense cities.

c)  To explain how density can enforce a sense of psychological distance.


d)  To demonstrate how cities become more and more dense with time.

Directions: Read the passages given below and answer the questions that follow.

Passage - 5

In a surprise development last week, the Government withdrew the Personal Data Protection (PDP) Bill, 2019,
thereby
abruptly halting the country's quest for a national data protection law that had been in the works for over
five years. The
reasons for the Government's decision are brief and cryptic. The short circular issued by the
Minister of Electronics and
Information Technology simply states that considering the report of the Joint Parliamentary
Committee (JPC) - it had proposed
81 amendments and made 12 recommendations - "a comprehensive legal
framework is being worked on". "In these
circumstances", the Government proposed to withdraw the Bill and
present a new Bill "that fits into the comprehensive legal
framework".

Interestingly, there is no elaboration on what such a "comprehensive legal framework" entails. The Government
could enact a
fresh privacy legislation or a comprehensive data protection law (covering both personal and
non-personal data). Alternatively,
it could subsume data protection under its ongoing attempts at revising the
existing Information Technology Act, 2000. It could
also enact a digital markets law, along the lines of the European
Union's Digital Services Act, focusing on competition and
innovation in the digital space. Unfortunately, the Ministry's
circular leaves us with no clarity on the way forward.

The Ministry's attribution of the withdrawal to the JPC Report is also at odds with the proposed amendments of the
JPC,
which did not recommend withdrawing the PDP Bill in favour of a comprehensive legal framework.

The lack of clarity is compounded by the fact that the circular does not establish any timelines on when the new Bill
will be
introduced in Parliament, or when it will be passed. This is particularly important, given the drafting history
of the PDP Bill.
When the Supreme Court of India affirmed the right to privacy in its historic K.S. Puttaswamy
judgment in 2017, the nine-judge
Bench of the Court referred to the Government's Office Memorandum constituting
the B.N. Srikrishna Committee to suggest a
draft Data Protection Bill. The committee released its draft Personal
Data Protection Bill in 2018, which was the first public
articulation of a data protection law in India.

Subsequently, when the Supreme Court upheld the constitutionality of the Aadhaar Act, the majority emphasised
that it
believed that "there is a need for a proper legislative mechanism for data protection". It "impressed" upon the
Central
government to bring out a "robust data protection regime" through the enactment of a law based on the
recommendations of
the Srikrishna Committee Report, with modifications as deemed necessary.
In December 2019, the Government introduced
the PDP Bill, 2019 in the Lok Sabha as a comprehensive personal
data protection regime. Considering the importance of the
Bill and the controversies associated with various
provisions, the Bill was referred to the JPC for its recommendations. In
2021, the JPC suggested multiple
amendments to its re-worded Data Protection Bill, 2021, which privileged state
exceptionalism over individual
privacy, while continuing to strictly regulate corporate action.

Now, after five years of hard work and three iterations of data protection legislation, the Government has wasted its
efforts to
protect our privacy.
Q 21.   Given the information in the passage, which among the following would be appreciated by the author?

i. The government comes out with a proper timeline for enacting new legislation on data protection.

ii. The government gives more clarity on why the draft bill on data protection had to be taken down.
iii. The government brings in experts from the data protection discipline to create a comprehensive legal
framework.

a)  Only i and ii b)  Only ii and iii c)  Only i and iii d)  All of the above

Q 22.   Out of the following, select the option that best captures the author's central message through the passage.

a)  With the withdrawal of the bill on data protection, years of hard work have been put to waste.

b)  The government must come up and account for the bill's withdrawal on personal data protection.

c)  Withdrawing the private data protection bill based on the reasons provided is criminal negligence.
d)  The government would do well to devise a timeline for enacting the law on personal data protection.

Q 23.   Choose the option that comes closest to the word 'cryptic'.

a)  Acumen b)  Enigmatic c)  Apricate d)  Burgeon

Q 24.   If the information in the given passage is accurate, which among the following can be inferred from the
passage?

a)  The author does not expect the government to introduce a new personal data protection law.

b)  The Central government has defied the orders of the Supreme Court with its withdrawal of the bill.

c)  The drafting history of the now withdrawn data protection bill can give some cues about the new law.

d)  All of the above

Q 25.   Which among the following has not been mentioned in the passage as a course that the government may
take post
the withdrawal of the PDP bill?

a)  The government could include provisions for data protection under its attempts to revise Information
Technology Act.
b)  The government could draft new legislation on data protection covering personal and non-personal data.

c)  A law could be enacted by including the provisions of and thereby replacing the Information Technology
Act of 2000.

d)  
The government could enact a law based on the European model, focusing on competition and innovation
in the digital space.

Directions: Read the passages given below and answer the questions that follow.

Passage - 6

One way to sum up postwar American politics is to say that conservatives try to stop liberals from breaking the
liberals' own
rules. The "rules" in this formulation are those of liberalism in the broadest sense: constitutional
principles, the rule of law,
rights-based protections. [1] "Liberal" regimes aren't supposed to impose a particular
understanding of the Good on their
citizens; they're meant to ensure local and individual freedoms and enable
citizens to figure out what the Good is for
themselves.

But some liberals-typically the highly educated and privileged sort-tend to forget they are liberals and try to define
righteousness for everybody. [2] They do this by reallocating citizens' wealth according to their own ideals, regulating
private
economic behavior, dictating to local communities how they should govern themselves, imposing protean
codes of correct
speech and behavior on everybody else, and so on. Conservatives, in this admittedly biased way
of putting it, are there to
stop liberals from indulging these illiberal impulses; to remind them, in other words, that
they are liberals, not potentates.

Not everyone considered themselves good liberals, of course. On the left, communists, socialists and other
radicals of the
20th century rejected liberalism as an invention of capitalism and Cold War ideology. [3] On the
right, a confederation of
Catholic intellectuals in the 1960s and '70s seeked to burst the bands of liberalism and
establish a more overtly moral political
order. More recently, the progressive left has largely given up on the idea
that everyone, including people whose opinions and
customs progressives find loathsome, deserves legal
protections. Among conservatives, a group of "postliberal" intellectuals,
resolutely traditionalist in religious outlook,
have proposed scrapping liberalism altogether by regulating markets and
expanding state power in ways
that shore up "communitarian" values: i.e., by doing what the political left has been doing for
70 years but in a
"conservative" way.

In debates over liberalism, the name of one political philosopher nearly always gets thrown around: John Locke
(1632-1704).
It was Locke who, in the "Two Treatises of Government," formulated a conception of the state that
derived its legitimacy from
the consent of the people, protected private property, and ruled through representational
government. [4] Locke wrote his two-
part book at a time of intense political volatility-perhaps around 1680, during
the reign of the restored monarchy of Charles II,
though the work was published in 1689, just after the Glorious
Revolution. The "Two Treatises" was among the first great
attempts to envision a form of government in which one
conception of the Good would not be permitted to traduce all the
others. Locke, it's fair to say, was the archetypal
liberal. The subject has particular relevance for Americans, who for many
generations have been taught that the
Founders were deeply influenced by Locke. Claire Rydell Arcenas gets to the question
of Locke's effect on the
Founders in "America's Philosopher: John Locke in American Intellectual Life," but her book isn't only
about Locke's
influence; mainly it chronicles Americans' remarkably consistent esteem for a 17th-century English philosopher
who never set foot in the New World. One thing is clear early on in Ms. Arcenas's story: Americans really did read
and admire
Locke. But his popularity in the New World derived initially, in the early and mid-18th century, from the
"Essay Concerning
Human Understanding."
Q 26.   Which among the following can be true about "Two Treatises of Government"?

a)  The "Two Treatises" formulated that one conception of the Good could not be allowed to slander all
others.
b)  
The "Two Treatises" made an effort to imagine a specific type of government in which one understanding
of what is Good
could not be permitted to belittle all other conceptions.
c)  
The "Two Treatises" were the first practical attempt to imagine a system of government in which one
understanding of what is
good could not be permitted to degrade other conceptions.

d)  All of the above

Q 27.   According to the passage, why is John Locke almost always discussed in the context of liberalism?

a)  Since it was Locke, who formulated the principles on which liberalism is based.

b)  Since John Locke is considered the father of modern-day liberalism.

c)  Since John Locke was the one who criticized the ideology of Conservatives.

d)  Since John Locke was the first person to implement the modern-day principles of Liberalism.

Q 28.   Out of the following, what could be inferred from the information given in the passage?

a)  Most liberals in America tend to define righteousness for everybody.

b)  There is a shortage of good liberals in the present day and age.

c)  Liberalism is incomplete without discussing the philosopher John Locke.

d)  Potentates follow beliefs that are likely to be contrary to that of liberals.

Q 29.   Which among the following is synonymous with 'archetypal'?

a)  Representative b)  Standard c)  Exemplary d)  Any of the above

Q 30.   Which among the following statements contains a grammatical error?

a)  [1] b)  [2] c)  [3] d)  [4]

Current Affairs Including General Knowledge


Passage - 1

The 2022 Nobel Prize in Physiology or Medicine is awarded to [1] for his discoveries concerning the genomes of
extinct
hominins and human evolution. The relationship between Homo sapiens and extinct hominins has long
been a topic of great
interest. Paleontology and archeology are important for studies of human evolution. Modern
DNA technology provides
opportunities to investigate our ancient past with more precision. However, due to extreme
technical challenges resulting from
degradation of DNA during tens of thousands of years and contamination from
micro-organisms and contemporary humans, it
was long questionable whether the analysis of archaic DNA from
extinct hominin forms would be possible. Through extensive
technological developments, [1] set new rigorous
standards in this challenging area and succeeded in obtaining the genome
sequence of our closest extinct relative,
the Neanderthal.

This was followed by his sensational discovery of another extinct hominin, the Denisova, entirely from genome
data retrieved
from a small finger bone specimen. [1]'s work further established that Homo sapiens had mixed
with Neanderthals and
Denisovans during periods of co-existence, resulting in introgression of archaic DNA in
present-day humans. Striking
examples of archaic gene variants that influence the physiology of present-day
humans have already been demonstrated in a
research field that is now highly dynamic. Through his groundbreaking
discoveries, [1] opened a new window to our
evolutionary past, revealing an unexpected complexity in the evolution
and ad-mixture of ancient hominins, as well as
providing the basis for an improved understanding of genetic
features that make us uniquely human.

Q 31.   The 2022 Nobel Prize in Physiology or Medicine is awarded to [1] for his discoveries concerning the genomes
of
extinct hominins and human evolution. Who among the following recipients has been redacted with [1] in
the passage above?

a)  Ragnar Sohlman b)  Svante Paabo c)  Giorgio Parisi d)  Erik Lindberg

Q 32.   Name the Indian-American astrophysicist who was awarded the 1983 Nobel Prize for Physics with William
A. Fowler
for "theoretical studies of the physical processes of importance to the structure and evolution of
the stars".

a)   b)   c)  Meghnad Saha d)  Har Gobind Khorana


Venkatraman Ramakrishnan Subrahmanyan
Chandrasekhar

Q 33.   The first person and the only woman to win the Nobel Prize twice is____________.

a)  Andrea Ghez b)  Marie Curie c)  Maria Goeppert Mayer d)  Donna Strickland

Q 34.   The first Nobel Prize in Physics was awarded to physicist Wilhelm Rontgen in recognition of the extraordinary
services
he rendered by the discovery of ______________.

a)  X-rays b)  Radio waves c)  Gama rays d)  Beta rays

Q 35.   Which of the following statements is Not true regarding the Nobel Prizes?

a)  
The Nobel Prizes are five separate prizes that are awarded to "those who, during the preceding year,
have conferred the
greatest benefit to Mankind, according to Alfred Nobel's will of 1895."
b)  Alfred Nobel was a Swedish chemist, engineer, and industrialist most famously known for the invention
of dynamite.

c)  Nobel Prizes were first awarded in 1906.

d)  
In 1968, Sveriges Riksbank funded the establishment of the Prize in Economic Sciences in Memory of
Alfred Nobel, to also be
administered by the Nobel Foundation.

Passage - 2

Distributed ledger technology (DLT) refers to the protocols and supporting infrastructure that allow computers in
different
locations to propose and validate transactions and update records in a synchronised way across a
network. The idea of a
distributed ledger - a common record of activity that is shared across computers in different
locations - is not new. Such
ledgers are used by organisations (eg supermarket chains) that have branches or
offices across a given country or across
countries. However, in a traditional distributed database, a system
administrator typically performs the key functions that are
necessary to maintain consistency across the multiple
copies of the ledger. The simplest way to do this is for the system
administrator to maintain a master copy of the
ledger which is periodically updated and shared with all network participants.

By contrast, the new systems based on DLT, most notably Bitcoin and Ethereum, are designed to function without
a trusted
authority. Bitcoin maintains a distributed database in a decentralised way by using a consensus-based
validation procedure
and cryptographic signatures. In such systems, transactions are conducted in a peer-to-peer
fashion and broadcast to the
entire set of participants who work to validate them in batches known as "blocks".
Since the ledger of activity is organised into
separate but connected blocks, this type of DLT is often referred to as
"blockchain technology".
Q 36.   Blockchain is a peer-to-peer _____________ distributed ledger technology that makes the records of any
digital asset
transparent and unchangeable.

a)  Secure b)  Popular c)  Decentralized d)  Demanding

Q 37.   Who among the following introduced the digital online cryptocurrency known as Bitcoin?

a)  Nick Szabo b)  Wei Dai c)  Hal Finney d)  Satoshi Nakamoto

Q 38.   Which of the following statements is/are true?

a)  
Bitcoin is a cryptocurrency that uses Blockchain technology, whereas Blockchain is the underlying
technology that powers
Bitcoin and is used in a variety of ways.
b)  A blockchain allows for peer-to-peer digital currency transfers without the use of middlemen such as
banks.

c)  Without the necessity of third-party intermediaries, blockchain may conduct user transactions.

d)  All of the above

Q 39.   If a hacker wanted to alter a blockchain, what percentage of the block copies would he have to alter?

a)  1% b)  25% c)  51% d)  100%

Q 40.   Which of the following statements is Not correct regarding the Crypto-Currencies?

a)  All crypto-currencies are based on Blockchain technology.

b)  Altcoins refer to a class of alternative crypto-currencies that were launched after the success of Bitcoin.
c)  Applications that depend on the basic features of blockchain can be developed without anybody's
permission.

d)  None of the above

Passage - 3

The Indian Space Research Organisation confirmed that the Mars Orbiter craft has lost communication with
ground station,
it's non-recoverable and the Mangalyaan mission has attained end-of-life. The ISRO gave an
update on the Mars Orbiter
Mission and the national meet held on September 27 to commemorate the MOM, on
the event of completion of its eight years
in the Martian orbit.

It was also discussed that despite being designed for a life-span of six months as a technology demonstrator,
the MOM has
lived for about eight years in the Martian orbit with a gamut of significant scientific results on Mars as
well as on the Solar
corona, before losing communication with the ground station, as a result of a long eclipse in
April 2022, the national space
agency said.

During the national meet, ISRO deliberated that the propellant must have been exhausted, and therefore, the
"desired altitude
pointing" could not be achieved for sustained power generation, ISRO said. "It was declared that
the spacecraft is non-
recoverable, and attained its end-of-life", an ISRO statement said. "The mission will be everregarded
as a remarkable
technological and scientific feat in the history of planetary exploration".
Q 41.   The Mars Orbiter Mission (MOM), also called Mangalyaan was launched on 5 November 2013 by the Indian
Space
Research Organisation (ISRO). The Mars Orbiter Mission probe lifted off from the Satish Dhawan
Space Centre, Sriharikota,
Andhra Pradesh, using a_____________.

a)  Geosynchronous Satellite Launch Vehicle Mark I (GSLV Mk I)


b)  Polar Satellite Launch Vehicle (PSLV) rocket C25

c)  Geosynchronous Satellite Launch Vehicle Mark III (GSLV Mk III)

d)  Polar Satellite Launch Vehicle (PSLV) rocket C45

Q 42.   The first Asian nation to reach Martian orbit and the first nation in the world to do so on its maiden attempt
is__________.

a)  China b)  Japan c)  India d)  USA

Q 43.   Aryabhata was India's first satellite named after the famous Indian astronomer. It was launched
on_____________.

a)  19 April 1975 b)  1 April 1967 c)  1 January 1980 d)  1 March 1972

Q 44.   Why is the planet Mars red in color?

a)  due to the presence of Copper oxide b)  due to the presence of Sodium oxide

c)  due to the presence of Calcium oxide d)  due to the presence of Iron oxide

Q 45.   Recently, which one of the following has inaugurated the world's first-ever facility dedicated to monitoring
space debris
and safeguarding assets in space?

a)  SpaceX b)  NASA c)  ISRO d)  JAXA

Passage - 4

India and Bangladesh should resolve all bilateral issues including the difference over the waters of the Teesta,
Prime Minister
Sheikh Hasina said. After holding bilateral talks with Prime Minister Narendra Modi, the visiting
leader described India as the
"most important and closest neighbour" that is bound with Bangladesh through 54
common rivers and four thousand
kilometres of border. The two sides signed seven agreements covering railways,
science and technology, space cooperation,
media, and water sharing. Welcoming Ms. Hasina, Prime Minister
Narendra Modi said, India will walk beside Bangladesh to
defeat the anti-1971 forces.

"The two countries had resolved many outstanding issues in the spirit of friendship and cooperation and we hope
that all
outstanding issues including Teesta Water Sharing Treaty would be concluded at an early date," said Prime
Minister Hasina
referring to the long standing riparian issue that has eluded resolution. The two sides however
made a significant beginning in
river water sharing by reaching an agreement on withdrawal of water from the
common border river Kushiyara that will supply
water to parts of lower Assam as well as Sylhet of Bangladesh.

Prime Minister Narendra Modi highlighted the values that made India and Bangladesh to put up a joint fight in the
war of 1971
and said, "In order keep the spirit of 1971 alive, it is necessary that we should confront those forces
that want to hurt our
common values.
Q 46.   Sheikh Mujibur Rahman was widely known as Bangabandhu who served as the first __________ of
Bangladesh.
a)   b)  President c)  Prime Minister d)  Home Minister
Governor General of
Banladesh

Q 47.   Sheikh Hasina is the daughter of the Sheikh Mujibur Rahman and currently serving as the Prime Minister of
Bangladesh. Sheikh Hasina is associated with which of the following political parties?

a)   b)   c)  Awami League d)  


Bangladesh Democracy Party Bangladesh Republican Party Bangladesh Nationalist Party

Q 48.   Sheikh Mujibur Rahman was assassinated on_____________.

a)  August 15, 1960 b)  November 15, 1975 c)  August 15, 1975 d)  November 15, 1960

Q 49.   The Kushiyara River water agreement signed between India and Bangladesh on September 6 2022 is the
first major
water sharing accord between the two friendly neighbours since the Ganga water treaty in 1996.
Kushiyara River is a
distributary of the Barak River which originates in the uplands of___________.

a)  Arunachal Pradesh b)  Tripura c)  Manipur d)  Assam

Q 50.   Which of the following constitutional amendment acts gave effect to the acquiring of certain territories by
India and
transfer of certain other territories to Bangladesh?

a)  The 98th Constitutional Amendment Act b)  The 99th Constitutional Amendment Act

c)  The 100th Constitutional Amendment Act d)  The 101st Constitutional Amendment Act

Passage - 5

India's Ministry of Home Affairs (MHA) outlawed the Popular Front of India (PFI) and eight affiliate organizations for
five years
under the Unlawful Activities (Prevention) Act (UAPA), 1967. The ban came close on the heels of raids
on PFI offices across
the country. Hundreds of its leaders have been arrested.

The PFI has been declared an "unlawful association." According to the gazette notification of the ban, "the PFI and
its
associates or affiliates or fronts have been indulging in unlawful activities, which are prejudicial to the integrity,
sovereignty
and security of the country and have the potential of disturbing public peace and communal harmony
of the country." Its
cadres have been "engaging in violent and subversive acts." It has carried out several
"cold blooded killings of persons
associated with organisations espousing other faiths, obtaining explosives to
target prominent people and places and
destruction of public property."

The government has alleged that the PFI has links with terror groups such as the Students Islamic Movement of
India (SIMI),
the Jamat-ul-Mujahideen Bangladesh (JMB), and the Islamic State of Iraq and Syria (ISIS). The PFI
and its affiliates "operate
openly as a socio-economic, educational and political organisation but, they have been
pursuing a secret agenda to radicalise
a particular section of the society working towards undermining the concept
of democracy," MHA said in a statement. The ban
on the PFI means that "its members will be criminalised and its
funds frozen."
Q 51.   Which of the following organizations was/were merged to form the PFI in 2007?

a)   b)  National Democratic Front c)  Manitha Neethi Pasarai d)  All of the above
Karnataka forum for dignity

Q 52.   What is the name of the political party affiliated to the PFI?

a)  Democratic National Party of India b)  Social National Party of India

c)  Social Democratic Party of India d)  Indian Social Democratic Party

Q 53.   Under which of the following legislations has the Students Islamic Movement of India been banned?
a)  Unlawful Activities Prevention Act, 1967 b)  Indian Penal Code, 1860

c)  National Security Act, 1980 d)  Central Industrial Security Act, 1968

Q 54.   Who among the following is a journalist arrested and charged under the Unlawful Activities Prevention Act,
1967 by
the Uttar Pradesh Police in 2020?

a)  Siddique Kappan b)  Rana Ayyub c)  Mohammed Zubair d)  Rupesh Kumar Singh

Q 55.   In August 2022, the Supreme Court granted bail to which of the following, a poet and activist arrested in
connection
with the Bhima Koregaon case?

a)  Father Stan Swamy b)  Perala Gopi c)  Khurram Parvez d)  P Varavara Rao

Passage - 6

The International Court of Justice has asked Russia to immediately halt its military operation in Ukraine, in a 13-2
decision
which means 13 voters were in favour of the direction to Russia, while 2 were against. Indian judge at the
UN court Justice
Dalveer Bhandari voted in favour of the majority, against Russia.

"The Russian Federation, pending the final decision in the case, must immediately suspend the military operations
it
commenced in the territory of Ukraine on February 24, 2022," the order of the UN court said.

The rulings of the International Court of Justice are binding but there have been cases where countries have
ignored them, as
ICJ has no direct means of enforcing its orders. Two judges who were against the direction were
Vice-President Kirill
Gevorgian (Russia) and Judge Sue Hanqin (China).

Justice Dalveer Bhandari is serving his second term at the World Court. In 2012, he was elected for the first term
which
continued till 2018. He was renominated by India and beat UK's nominee Justice Greenwood to win another
term at the ICJ.
Q 56.   Who among following judges of India did not serve at the International Court of Justice (ICJ)?

a)  R S Pathak b)  Nagendra Singh c)  Sir Benegal Rau d)  M. Patanjali Sastri

Q 57.   Which of the following statements is Not true regarding the International Court of Justice (ICJ)?

a)  It was established in 1949 by the United Nations charter and started working in April 1950.
b)  Unlike the six principal organs of the United Nations, it is the only one not located in New York.

c)  Unlike the International Court of Justice (ICJ), the ICC is not part of the United Nations system.

d)  The judgment of ICJ is final and binding on the parties to a case and without appeal.

Q 58.   How many times did India vote against Russian positions during the Ukraine conflict till October 15, 2022 at
the United
Nations Organization?

a)  Two times b)  Three times c)  Four Times d)  Five times

Q 59.   Who among the following is the current India's permanent representative to the United Nations?

a)  Asoke Kumar Mukerji b)  Ruchira Kamboj c)  Vinay Mohan Kwatra d)  Syed Akbaruddin

Q 60.   Which of the following statements is Not true regarding the International Criminal Court (ICC)?

a)  
It is the first and only permanent international court with jurisdiction to prosecute individuals for the
international crimes of
genocide, crimes against humanity, war crimes and the crime of aggression.
b)  
The ICC lacks universal territorial jurisdiction and may only investigate and prosecute crimes committed
within member states
crimes committed by nationals of member states, or crimes in situations referred
to the Court by the United Nations Security
Council.
c)  Its founding treaty, the Paris Statute, entered into force on July 1, 1980.

d)  India is not a party to Rome Statute along with US and China.

Passage - 7

"Europe faces "unprecedented risks" to its natural gas supplies this winter after Russia cut off most pipeline
shipments and
could wind up competing with Asia for already scarce and expensive liquid gas that comes by
ship," the International Energy
Agency (IEA) said.

The Paris-based IEA said in its quarterly gas report released that European Union (EU) countries would need to
reduce use
by 13% over the winter in case of a complete Russian cut-off amid the war in Ukraine. "Much of that
cutback would have to
come from consumer behaviour such as turning down thermostats by one degree and
adjusting boiler temperatures as well
as industrial and utility conservation," the group said.

The EU on September 30 agreed to mandate a reduction in electricity consumption by at least 5% during peak
price hours.
Just a trickle of Russian gas is still arriving in pipelines through Ukraine to Slovakia and across the
Black Sea through Turkey
to Bulgaria. Two other routes, under the Baltic Sea to Germany and through Belarus and
Poland, have shut down.
Q 61.   In September 2022, the members of the _________have agreed to impose a price cap on Russian oil in a
bid to hit
Russia's ability to finance the war in Ukraine.

a)  G7 b)  G20 c)  G4 d)  G77

Q 62.   On 31 August, 2022, Russia stopped the flow of gas via the Nord Stream 1 pipeline citing maintenance
issues and the
need to carry out repairs. Nord Stream is a pair of offshore natural gas pipelines in Europe
that runs under the
__________from Russia to Germany.

a)  Baltic Sea b)  Red Sea c)  Caspian Sea d)  Black Sea

Q 63.   The Yamal-Europe natural gas pipeline is a 4,107-kilometre-long pipeline connecting ___________ natural
gas fields
in the Yamal Peninsula and Western Siberia with Poland and Germany, through Belarus.

a)  Russia b)  Finland c)  Norway d)  Iran

Q 64.   The European Union's "Green Deal" seeks to make Europe carbon-neutral by ________and reduce emissions
by at
least 55% by 2030.

a)  2030 b)  2045 c)  2050 d)  2070

Q 65.   Europe is struggling to contain an energy crisis that could lead to rolling blackouts, shuttered factories and
a deep
recession. The primary cause is _____________.

a)  
USA has choked off the supplies of cheap natural gas that the continent depended on for years to run
factories, generate
electricity and heat homes.
b)  
UK has choked off the supplies of cheap natural gas that the continent depended on for years to run
factories, generate
electricity and heat homes.

c)  
Russia has choked off the supplies of cheap natural gas that the continent depended on for years to run
factories, generate
electricity and heat homes.
d)  
Norway has choked off the supplies of cheap natural gas that the continent depended on for years to run
factories, generate
electricity and heat homes.

Legal Aptitude
Directions for questions 66 to 105: You have been given some passages followed by questions based on each
passage.
You are required to choose the most appropriate option which follows from the passage.
Only the information given in the
passage should be used for choosing the answer and no external knowledge of
law howsoever prominent is to be applied.

Passage - 1

To commit the tort of negligence, there are primarily 6 main essentials that are required namely -

(i) Duty Of Care


It is one of the essential conditions of negligence in order to make the person liable.
It means that every person owes, a duty
of care, to another person while performing an act. Although this
duty exists in all acts, but in negligence, the duty is legal in
nature and cannot be illegal or unlawful and
cannot be of moral, ethical or religious nature.

(ii) The Duty must be towards the plaintiff


A duty arises when the law recognizes a relationship between the defendant and the
plaintiff and requires
the defendant to act in a certain manner toward the plaintiff. It is not sufficient that the defendant owed a
duty
of care towards the plaintiff but it must also be established which is usually determined by the judge.

(iii) Breach of Duty to take care


It's not enough for a plaintiff to prove that the defendant owed him a duty of care but he must also establish
that the defendant
breached his duty to the plaintiff. A defendant breaches such a duty by failing to exercise
reasonable care in fulfilling the duty.

(iv) Actual cause or cause in fact


In this scenario, the plaintiff who is suing the defendant for negligence has the liability to prove is that the
defendant's violation
of duty was the actual cause of the damages incurred by him.

(v) Proximate cause


Proximate cause means "legal cause," or the cause that the law recognizes as the primary cause of the
injury. It may not be
the first event that set in motion a sequence of events that led to an injury, and it may not
be the very last event before the
injury occurs.

(vi) Consequential harm to the plaintiff


Proving that the defendant failed to exercise reasonable care is not enough. It should also be proved that the
failure of the
defendant to exercise reasonable care resulted in damages to the plaintiff to whom the defendant
owed a duty of care.

However, there are several defences available for the suit against the tort of negligence. These are:

(i) Contributory negligence by the plaintiff


Contributory negligence means that when the immediate cause of the damage is the negligence of the
plaintiff himself, the
plaintiff cannot sue the defendant for damages and the defendant can use it as a defence.

(ii) An Act of God


An Act of God is a direct, violent and sudden act of nature which by any amount of human foresight could
have been foreseen
and if foreseen could not by any amount of human care and skill have been resisted.

(iii) Inevitable Accident


An inevitable accident can also be called as a defence of negligence and refers to an accident that had no
chance of being
prevented by the exercise of ordinary care, caution, and skill. It means a physically unavoidable
accident.
Q 66.   A was a driver of army truck and was supposed to deliver grocery items to the cantonment. On night,
while he was on
his way to the cant, he hits a passer-by. Later it was revealed that due to the heavy rains,
the driver was unable to control the
truck. The passer-by filed a suit of negligence against the truck driver.
Decide.

a)  The suit would fail because the passer-by was hit by way of an inevitable accident.

b)  The suit would fail because the passer-by has contributed to the negligence.
c)  The suit would succeed because, the driver was speeding.

d)  The suit would succeed because the driver failed to take due care while driving in the rain.

Q 67.   A bridge connecting two of the most populous parts of the city, was 50 years old. It was the busiest bridge
and was not
regularly repaired for wear and tear. One day, A who was to make a world record for the longest
bicycle, came to the bridge
with the bicycle that weighed 1000 kgs. However, after few minutes, the bridge
collapsed resulting in severe injuries to the
people present. Later A filed a suit for negligence. Decide.

a)  The suit would succeed because 1000kgs is not that a heavy weight to impact a well-built bridge.

b)  The suit would succeed because the concerned authorities committed a breach of care.
c)  The suit would fail because 1000kgs is a heavy weight and hence it was contributory negligence.

d)  The suit would fail because the bridge would not have collapsed, but for the heavy bicycle.

Q 68.   During one of the martial art performance, A who was trained at using sword to perform the act, hit B due to
which he
was bleeding profusely from his neck. A woman in the audience had a cardiac arrest at the ghastly
sight. She filed a suit for
negligence against A. Decide.

a)  The suit would succeed because A breached the duty to care.

b)  The suit would succeed because A has a duty towards the spectators.

c)  The suit would fail because cardiac arrest is not the proximate effect of the act of A.
d)  The suit would fail as it would be a case of volenti non-fit injuria.

Q 69.   Which of the following is the correct statement with respect to proximate cause being one of the essentials
for the tort
of negligence?

a)  Proximate cause needs to be last act.

b)  Proximate cause refers to an intervening event before the primary cause.

c)  Proximate cause is the first event that set the injury in motion.
d)  Proximate cause is the foreseeable event resulting in injury.

Q 70.   Which of the following is not an essential condition for negligence?

a)  Consequential harm to the plaintiff is an essential condition under negligence.

b)  Duty of care towards the plaintiff is an essential condition.

c)  Actual cause or cause in fact by the defendant


d)  The duty must be towards the defendant

Directions for questions 66 to 105: You have been given some passages followed by questions based on each
passage.
You are required to choose the most appropriate option which follows from the passage.
Only the information given in the
passage should be used for choosing the answer and no external knowledge of
law howsoever prominent is to be applied.

Passage - 2

Article 22 constituted within the right to freedom is one of the parts of the fundamental rights guaranteed under the
constitution. This article is covered in two major parts, protection and rights granted in case of arbitrary arrest also
known as
punitive detention, and safeguards against preventive detention. The main difference is that whether a
person is charged with
a crime or not. Article 22(1) states that any person who is arrested, cannot be detained in
custody without being informed of
the grounds of any such arrest as soon as possible, the arrested person has
the right to consult a legal practitioner at all times
and be defended by a lawyer of his own choice. This right is
expanded right from the moment of the person's arrest. Article
22(2) ensures the right of the accused to be
produced before a magistrate. When a person is arrested, the person or police
officer making the arrest should
bring the arrested person before a magistrate or judicial officer within 24 hours, without any
unnecessary delay,
this provision helps to keep a check on the investigation of the police regarding the matter at hand. It
protects the
accused from being trapped into wrongful detention. Clause 3 of the Article 22 clearly states that none of the
rights
mentioned in clause 1 and 2 of the Article would be applicable for a person who is deemed to be an enemy alien
and
anybody who is arrested or detained under the law providing for preventive detention. A person can be put in
jail/custody for
two reasons. One is that he has committed a crime. Another is that he has the potential to commit a crime in the future. The
custody arising out of the latter is preventive detention and in this, a person is deemed
likely to commit a crime. Thus,
Preventive Detention is done before the crime has been committed. Clause 5 of
Article 22 states that the ground of detention
should be communicated to the detenu within minimum five or a
maximum of fifteen days. In no case should it be delayed
beyond fifteen days. It must be completely furnished to
the detenu, including all the facts and should not be only bare recital of
the grounds. Any lapse within this provision
would render the detention order void. Clause 6 of the article stands an exception
to clause 5 and states that the
detaining authority is not mandatorily required to disclose any such facts which it considers to
be against the
public interest to disclose.
Q 71.   A was arrested at 10 P.M. for possessing narcotic drugs and other psychotropic substances beyond the
permissible
limit. While on the way to the nearest magistrate, who was 80 km away, the police jeep met with
many religious processions
and consequently heavy traffic. Finally, at 11:30 pm the next day, A was finally
presented before the magistrate. A claims that
his right under 22(1) is violated. Decide.

a)  A's right has not been violated because travel time is excluded while considering the 24 hour cap.

b)  A's right has not been violated because he was arrested under preventive detention clause.

c)  A's right has been violated because he was presented before the magistrate after 24 hours of the arrest.
d)  A's right has been violated because he was not given the permission to contact a legal practitioner at the
time of arrest.

Q 72.   A was in an art exhibition when he was arrested by the police. On asking the reason for such arrested,
the police
refused to divulge any information about the arrest. Choose the correct statement.

a)  Action of the police officer would be correct if the information is against the public interest.

b)  Action of the police officer would be correct if the information does not affect the detention of the accused.
c)  Action of the police is fair as per article 22(1).

d)  Action of the police officer would be unfair if the detention is made under 22(3)(b).

Q 73.   A was a daily wage labourer, who had difficulty in providing two meals a day to his family. The pandemic had
made the
situation worse for him. One day, one of the contractors accused him of stealing the left-over
supplies and selling them in the
black market. The police came and arrested him on the ground of suspicion.
Being from the lower economic strata, it was
impossible for him to engage a legal help. What remedy does
the constitution of India, provide for him?

a)  A would be given opportunities to earn, so that he could engage a lawyer.

b)  A would be provided with a lawyer, chosen by the prison authorities.


c)  A would be given a lawyer of his choice.

d)  A would be allowed to defend himself in the court of law.

Q 74.   Which of the following is correct with respect to preventive detention?

a)  Preventive detention is a punitive method for suspects.


b)  Preventive detention necessitates trial.

c)  Police cannot make arrest without warrant under preventive detention.


d)  Preventive detention is a preventive remedy.

Q 75.   In what situation can the detention be extended beyond 3 months?

a)  Under no circumstances, should the detention be extended beyond 3 months.


b)  Detention could be extended beyond 3 months if the advisory board gives sufficient reason for the same.

c)  Detention could be extended beyond 3 months if the accused is charged with murder.

d)  Under the Criminal Procedural Code, the detention could be extended beyond 3 months.

Directions for questions 66 to 105: You have been given some passages followed by questions based on each
passage.
You are required to choose the most appropriate option which follows from the passage.
Only the information given in the
passage should be used for choosing the answer and no external knowledge of
law howsoever prominent is to be applied.

Passage - 3

When one party delegates some authority to another party whereby the latter performs his actions in an independent
fashion,
on behalf of the first party, the relationship between them is called an agency. The Indian Contract Act,
1872 defines an
'Agent' in Section 182 as a person employed to do any act for another or to represent another in
dealing with third persons.
According to Section 182. The person for whom such act is done, or who is so
represented, is called the "principal".
Therefore, the person who has delegated his authority will be the principal.
An agency can be created by: Direct (express)
appointment, Implication, situation of necessity where one person
can act on behalf of another to save the person from any
loss or damage, without expressly being appointed as an
agent, creation by estoppels wherein one person behaves in such a
manner as to make someone believe that he
is an authorized agent on behalf of someone, by way of ratification in which a
person who acted as another
person's agent without his knowledge is later ratified by that person. Generally, there exists no
agency between a
husband and wife, except in cases where it has expressly or impliedly been sanctioned that either of them
would
do certain acts or transactions as the agent of the other. It is necessary that the husband and wife are living
together in
a domestic establishment. In cases where the husband and wife live separately due to the husband's
fault, the husband is
required to pay for the necessaries to his wife, resulting in an agency of necessity. A relationship
of agency can come into
existence between the two through contract, appointment, or ratification. Authority of an
agent can be either expressed or
implied. In the former, the authority is given by words spoken or written, in the
latter the authority is inferred from facts and
circumstances. Implied authority is of four main types: Incidental
authority- doing something that is incidental to the due
performance of express authority, usual authority- doing
that which is usually done by persons occupying the same position,
customary authority- doing something according
to the pre-established customs of a place where the agent acts,
circumstantial authority- doing something according
to the circumstances of the case.
Q 76.   A is the owner of a truck and B was its driver. A lived in Delhi, however the truck operates between the state
of
Maharashtra and Andhra Pradesh. One day, B stopped at a petrol pump, and told that the payment would
be made by A.
Later A refused to pay. Decide the validity of A's action.

a)  A must pay because B was under the implied authority to fill the petrol.
b)  A must pay because B had express authority over the upkeep of the truck.

c)  A must not pay because B had not authority to fill the petrol.

d)  A must not pay because B did not seek prior permission from A.

Q 77.   A had a shoe manufacturing factory at Calcutta, however he lived in Delhi. To manage the affairs of the
factory, A has
appointed B. In the recent years, the leather industry has been facing slowdown, the impact of
which could be felt on A's
factory as well. Due to which this time, instead of full grain leather, which was
higher in cost and would have severely
disturbed the profit of the company, B bought bonded leather for
operations. However, the retailers who was sold this batch of
shoes, complained of the degraded quality and
demanded compensation for loss from A. Decide.
a)  A should compensate the retailers because he is the owner of the manufacturing unit.

b)  A should compensate the retailer because B acted in his capacity while managing the factory.
c)  A need not compensate the retailers because he did not give B, the authority to deal with lower quality
leather.

d)  A need not compensate the retailers because he did not manage the manufacturing unit.

Q 78.   A was a professor in a university and lived with his wife in the university dormitory for staff. One day, the wife
bought
some groceries on credit from B, who owned a shop inside the university. B now demanded A to fulfil
the debt. Decide.

a)  A had to repay the debt, as there is a natural relationship of agency between husband and wife.
b)  A had to repay the debt, as the husband must fulfil the domestic necessities of wife.

c)  A need not repay the debt, as husband and wife did not live in a domestic establishment.

d)  A need not repay the debt, as there is no express authority between husband and wife.

Q 79.   Which of the following is the necessary condition to become an agent?

a)  An agent must be of Indian origin. b)  An agent must be a literate.

c)  An agent must be appointed by a literate. d)  An agent must be above 18 years of age.

Q 80.   Which of the following is not a method for the creation of agency?

a)  In case of necessity, one person can act on behalf of other, to save losses, danger etc.

b)  A person could act as an authorised person in front of a third party.


c)  The principal could ratify the act of a person as his agent, after the act has been done.

d)  The principal could put a person in a position to act as an agent, against his will.

Directions for questions 66 to 105: You have been given some passages followed by questions based on each
passage.
You are required to choose the most appropriate option which follows from the passage.
Only the information given in the
passage should be used for choosing the answer and no external knowledge of
law howsoever prominent is to be applied.

Passage - 4

Malicious prosecution is the malicious intention of unsuccessful criminal or bankruptcy or liquidation proceedings
against
another without reasonable or probable cause. Generally, it can be said that the malicious prosecution is
defined as a judicial
proceeding instituted by one person against another, from wrongful or improper motive,
without any reasonable and probable
cause to justify it. "A malicious prosecution consists in maliciously causing
process to be issued, whereas an abuse of
process is the employment of legal process for some purpose other
than that which it was intended by the law to affect the
improper use of a regularly issued process."

Following are the essential elements which the plaintiff is required to prove in a suit for damages for malicious
prosecution: -
prosecution by the defendant, absence of reasonable and probable cause, defendant acted
maliciously, termination of
proceedings in the favour of the plaintiff, plaintiff suffered damage as a result of the
prosecution. The damages are commonly
grouped as three- the damage to man's fame, the damage done to a
person, the damage done to person property. In the
damage of man's fame the matter where the person is
accused without reason because of that, it will cause general public
outrage by a perceived offence against
morality. Under the damage done to the person put in danger of losing his life and
liberty. In the case of damage to
person and property, he is forced to pay expenses of litigation to acquit himself of the crime
which he is accused.
In the false imprisonment, they will control the personal liberty of someone without any lawful
justification.
But when it comes to malicious prosecution the damage is basically an abuse of the legal process. By the action
of a private individual legal action, the plaintiff liberty is wrongly controlled in false imprisonment but in malicious
prosecution
with help judicial sanction, they will arrest the plaintiff.
Directions for questions 66 to 105: You have been given some passages followed by questions based on each
passage.
Q 81.   A and B worked in the office. A complained that B, passed casteist slurs against him. A departmental enquiry
was set
up against B, in which he was found to be innocent. Later B, instituted a suit of malicious prosecution
against A. Decide.

a)  B would not succeed as departmental enquiry cannot be equated judicial proceeding.

b)  B would not succeed as he did not incur any cost in the legal proceeding.

c)  B would succeed as A filed a false complaint against him.


d)  B would succeed as he was held innocent in the departmental enquiry.

Q 82.   Z filed a suit of defamation against Y. However, the court held that the statements made by Y were not
defamatory in
nature, therefore the case was dismissed. Y instituted a case of malicious prosecution against
Z. Decide.

a)  Z's suit would fail because there was not judicial proceeding.

b)  Z's suit would fail because Y did not have any malicious intent.

c)  Z's suit would succeed because the suit of defamation was dismissed in his favour.

d)  Z's suit would succeed because his reputation was lowered in the eyes of the right-thinking people of the
society.

Q 83.   R entered into a contract with P to deliver a specified amount of goods to R within a stipulated time.
P refused to fulfil
the contractual obligation, asking for higher price of the supply as the market price has
soared due to shortage of raw
material. P filed a case of breach of contract against R, the court held R to be
guilty. R later instituted a case of malicious
prosecution against P. Decide.

a)  R would succeed, because the court had held him innocent.

b)  R would succeed, because it was a case of force majeure.


c)  R would fail, because malicious prosecution could be instituted in civil cases.

d)  R would fail, because he was found to be guilty of the breach of contract

Q 84.   Which of the following is not an essential for malicious prosecution?

a)  Prosecution by the defendant b)  Absence of reasonable and probable cause

c)  Defendant acted in good faith d)  Plaintiff suffered damages as a result of the prosecution.

Q 85.   Which of the following statement is not correct with respect to the tort of malicious prosecution?

a)  Malicious prosecution includes criminal proceeding by way of appeal and revision.

b)  Institution of the case, on the order of the court, based on the false evidence given by the defendant is no
defence.
c)  Matter before the executive authority would not amount to malicious prosecution.

d)  Malicious prosecution includes the employment of legal process for any cause, which would establish its
improper use.

You are required to choose the most appropriate option which follows from the passage.
Only the information given in the
passage should be used for choosing the answer and no external knowledge of
law howsoever prominent is to be applied.

Passage - 5

In layman's language it implies the use of otherwise unlawful actions in order to protect oneself or any other
individual, to
protect property or to prevent any other crime. Section 96 to 106 of Indian Penal Code 1860 contains
the provisions regarding
the right of private defense available to every citizen of India. This right can only be
exercised in case of overhanging danger
and state aid or help is not available. . This right has basically evolved
with the time by judgments and decisions of the
Supreme Court of India. One of the most important principles of
private defense is the 'reasonableness' of the defense used.
In most common language it implies the use of
generally or otherwise unlawful actions in order to protect oneself or any other
individual, to protect property or to
prevent any other crime. Simply it can be termed as any action done in the course of self-
protection. According to
Article 51(a)(i) of Indian Constitution the State is having a fundamental duty of the state to protect
public property
and abjure violence. It implies that it is the fundamental duty of the state to protect its citizens and their
property
from any harm, and in case the aid or help of state is not available and the danger is overhanging and is unavoidable
at the moment then the person is authorized to use his force to protect himself from any harm or injury. Sec 97 of
IPC states
that every citizen is having this right subject to certain restrictions (mentioned in sec 99) to defend his
own body or body of
any other person, against; any offence affecting to the human body; the property whether
immovable or movable, of himself or
of any other person, against any act, which is an offence falling under the
definition of robbery, theft, mischief, criminal
trespass or which is an attempt to commit theft, robbery, mischief or
criminal trespass. It was granted as a right for self-
protection to every citizen of India but it is often misused by
many people by treating it as an excuse of committing any crime
or offence. It is a right granted for defence and not
for vengeance and may not be used as a measure of taking revenge. This
right of private defence is not available
against any lawful action i.e. when the actions of a person are lawful and not resulting
into any offence the right of
private defence can't be utilized. Sometimes some people provoke others to act in aggression and
use it as an
excuse for the harm caused and even murder. But this can't be used in a situation where the aggression was
shown by the accused only. It is treated as license to kill by many people as the IPC is not in clear on the situation
where an
attack may be provoked as pretence of killing. But the court has asserted that the private defence is
available only to the
persons who act in good faith and don't misuse it as an excuse to justify their unlawful act or
act of aggression. The extent of
the right of private defence and the limitations in the exercise of this right may be
summarized as below: -

(1) There is no right of private defence against an act which is not in itself an offence under this Code. This does
not cover the
case of exceptions.

(2) The right commences as soon as and not before a reasonable apprehension of danger to the body arises
from an attempt
or threat to commit some offence. The right is availed of only against a danger imminent,
present and real.

(3) It is defensive and not a punitive or retributive right. In no case the right extends to the inflicting of more harm
than it is
necessary to inflict for the purpose of defence, though reasonable allowance should be made for
bona fide defender.

(4) The right extends to the killing of actual assailant when there is a reasonable and imminent danger of the
atrocious crimes
enumerated in the six clause s of Section 100.

(5) There must be no safe or reasonable mode of escape by retreat, for the person confronted with an impending
peril to life
or of grave bodily harm except by inflicting death on the assailant.

(6) The right being, in essence, a defensive right, does not accrue and avail where there is time to have recourse
to the
protection of public authorities.
Q 86.   P, a constable came to S's house to arrest him on the suspicion of kidnapping his neighbour's nephew.
P was resisting
and therefore S hit him with a stick. However, furious by this, P retaliated by attacking S,
on his head by a brick lying on the
floor. Later P pleaded private defence.

a)  P is liable as private defence does not lie against a legal act.

b)  P is liable as the force applied is disproportionate.

c)  P is not liable as he was exercising his right of self-defence.


d)  P is not liable because S refused to show him the arrest warrant.

Q 87.   M was working in his field late night, when the thieves came to steal his fresh harvest. When the thieves saw
M
working, they ran away. M, to protect his harvest, shot one of the thieves on his leg. Decide.

a)  M would be liable because the force used was disproportionate.

b)  M would be liable because the need of the force was not immediate and imminent.

c)  M would not be liable because he was protecting his harvest from thieves.
d)  M would not be liable because had he not shot them, thieves would have attempted to steal again.

Q 88.   G was very fond of the garden in his courtyard, which was home to many birds and animals like squirrels
and rabbits
etc. However recently many children have started playing in the nearby park, and due to the loud
noises, birds do not come to
G's courtyard. G was very upset with this. One day a ball landed on G's
courtyard, when a child came to take the ball, G came
furious with a sword and struck on the child's hand.
Decide.

a)  G would not be liable because he was protecting the bird in his courtyard.
b)  G would not be liable because he was exercising his right to defence of his property.

c)  G would be liable because the force applied was disproportionate.


d)  G would be liable because no self-defence lies against children.

Q 89.   Which of the following is the incorrect statement with respect to right to private defence?

a)  There is no right to defence against an act which is not an offence.

b)  The right is preventive in nature and not punitive or retributive.

c)  The right could be availed even if there is an alternative to escape.

d)  The right can be availed even if there is time to take help from the public authorities.

Q 90.   Which of the following statement, correctly summarises the nature of the right of self-defence?

a)  Self-help is the first principle of the right of private defence.


b)  Protecting the body and property of other is charity.

c)  Right to private defence would not lie against the person of unsound mind.

d)  Right to private defence would apply to torts as well.

Directions for questions 66 to 105: You have been given some passages followed by questions based on each
passage.
You are required to choose the most appropriate option which follows from the passage.
Only the information given in the
passage should be used for choosing the answer and no external knowledge of
law howsoever prominent is to be applied.

Passage - 6

Defamation in law, is attacking another's reputation by a false publication (communication to a third party) tending
to bring the
person into disrepute. Generally, defamation requires that the publication be false and without the
consent of the allegedly
defamed person. Words or pictures are interpreted according to common usage and in
the context of publication. Injury only
to feelings is not defamation; there must be loss of reputation. The defamed
person need not be named but must be
ascertainable. A class of persons is considered defamed only if the
publication refers to all its members particularly if the class
is very small- or if members are specially imputed.
Actual truth of the publication is usually a defence to a charge of
defamation. Legal privilege arising from a special
relationship or position also relieves liability. Defamation is the publication of
a statement which reflects on a
person's reputation and tends to lower him in the estimation of right-thinking members of
society generally or
tends to make them shun or avoid him. Section 499 of the IPC defines 'defamation' as being committed:
A. Through: (i) words (spoken or intended to be read), (ii) signs, or (iii) visible representations; B. Which: are a
published or
spoken imputation concerning any person; C. If the imputation is spoken or published with: (i) the
intention of causing harm to
the reputation of the person to whom it pertains, or (ii) knowledge or reason to believe that the imputation will harm the
reputation of the person to whom it pertains will be harmed. Under criminal law on
the other hand merely proving that the
statement was true is not a good defense and besides this, the defendant
must show that it was made for public good also.
Making a fair comment on matters public interest is a valid
defense to an action for defamation. Another defence for criminal
defamation is priviledge namely absolute
(parliamentary and judicial) and qualified. As far as defamation under tort law is
concerned, as a rule, the focus is
on libel (i.e., written defamation) and not on slander (i.e., spoken defamation). In order to
establish that a statement
is libellous, it must be proved that it is (i) false, (ii) written; (iii) defamatory, and (iv) published. It is
significant to
mention that a defamation bill was proposed by the Rajiv Gandhi government to deal with the law pertaining to
defamation. However, Defamation Bill, 1988 received widespread criticism from the media and opposition parties
due to its
draconian provisions; as a result, it was withdrawn.
Q 91.   Mr. X, lost a limb in an accident 10 years ago. He faced lot of difficulties initially, however with utter determination
and
resilience he has successfully moved past all the problems and is now the coach of inter-state hockey
team. One day, right
after the team selection for the interstate tournament, one of the players came furious
and started shouting in the hockey field
"this slick lame is the torch bearer of favouritism in the team, he only
places his pets in the team." Is the player liable for
defamation?
a)  The player is not liable for defamation, because he did not take any name.

b)  The player is not liable for defamation, because X was not on the field.
c)  The player is liable for defamation, because whatever he said could be ascertainable to Mr X.

d)  The player is liable for defamation, because he could be heard clearly on the field.

Q 92.   During one of the parliamentary debates, Mrs N, an MP called PM a thief and a traitor, for stealing the public
money
and not fulfilling the promises made to the citizens. However, the people of the ruling party were not
amused and filed a
complaint for defamation against Mrs N. in the court of law. Decide whether the complaint
is valid.

a)  Mrs. N would be liable for defamation because she has lowered the reputation of PM.

b)  Mrs. N would be liable for defamation because privileges do not extend to the PM of the country.

c)  Mrs. N would not be liable for defamation because she has spoken the truth.
d)  Mrs. N would not be liable for defamation because she is protected under parliamentary privilege.

Q 93.   Which of the following correctly explains the offence of libel with respect to the information given in the
passage?

a)  Libel refers to spoken defamation by the offender against the petitioner.

b)  Any written defamation by the offender against the petitioner is libel.

c)  Any defamation by the offender against the petitioner in published form is libel.

d)  A defamation which is false and does not implicate the offender is libel.

Q 94.   Which of the following is not a defence for criminal defamation?

a)  A fair comment made is a defence under criminal defamation.

b)  Truth is a defence for criminal defamation

c)  Several privileges, both qualified and absolute, is defence under criminal defamation

d)  Publication made of the defamatory statement is a defence under criminal defamation.

Q 95.   A wrote a letter to B, with a message that "I know how you cleared the paper and got into the position that you
are
now, I saw you lurking and passing chits in the examination hall. I think it is time that a charlatan like you
should resign." The
truth however was that A mistook B to be M. Decide the liability of A.

a)  A is liable because the statement he made about B was not true.

b)  A is liable because the letter is read by everyone in the postal department.

c)  A is not liable because it was a fair comment based on his opinion.

d)  A is not liable because the letter is not read by anyone except B.

Directions for questions 66 to 105: You have been given some passages followed by questions based on each
passage.
You are required to choose the most appropriate option which follows from the passage.
Only the information given in the
passage should be used for choosing the answer and no external knowledge of
law howsoever prominent is to be applied.

Passage - 7

The Constitution of India, under Article 32 and Article 226, provides five types of writs to the citizens of India. These
writs have
different meanings and applications. They are:

Habeas corpus: The term habeas corpus means 'you may have the body of'. This writ can be used when a person
is illegally
detained. Using this writ, the court directs the person detained to be brought before the court to examine
the legality of his
detention. A writ of habeas corpus can be filed under the following circumstances- When a
person is detained but is not
produced before the magistrate within 24 hours of arrest, excluding the time consumed
in travelling. When a person is
arrested even when he has not violated any law. When a person is arrested with a
malafide intention.

Mandamus: The term mandamus means 'we command'. It is generally a command which is issued by the court
to a public
servant asking him to perform his duty, which he has failed to do so. The writ of mandamus can also be
issued against a
corporation, an inferior court or a government body for the same reason, i.e., they failed to
perform their official duties.

Certiorari: The term certiorari means 'certified or to be informed'. It is generally issued by a higher court to a lower
court either
to transfer the pending case to a higher authority or squash the judgement passed. In the year 1991,
the Supreme Court ruled
that the writ of certiorari can also be issued against administrative authorities if their
judgements are violating the rights of an
individual.

The writ of certiorari can be issued under the following conditions: There must be a court that has the authority or
have a legal
right to act judicially. If the judgement by any inferior court is against the laws mentioned under the
Constitution. If the
judgement given by a lower court contains an error.

Prohibition: The term prohibition simply means 'to forbid'. The writ of prohibition can only be issued against judicial
and quasi-
judicial authorities. This writ is generally not available against administrative authorities and private
individuals. It can be
issued against a lower court or by a superior court to forbid the act which is performed outside
its jurisdiction at any stage of
its proceedings.

Quo-warranto: The term quo- warranto means 'by what authority or by what warrant'. It is generally issued by a
court to
enquire the legality or under what authority he is holding that office. It prevents the illegal holding of a public
office by any
person.
Q 96.   The writ petition is issued under which provision of the constitution?

a)  Article 32 of the constitution provides for writ petition

b)  Article 13 of the constitution, mentions about writ petition

c)  Article 45 of the constitution is about writ petition to the Supreme Court

d)  Article 37 of the constitution provides for writ petition to the High Court.

Q 97.   Which of he following statements is not correct with respects to the power of writs?

a)  Article 226 has a wider ambit than article 32.

b)  Article 226 is referred when seeking remedy under high court.

c)  Article 226 is restricted to get remedy against violation of fundamental rights.

d)  Article 226 provides remedy against violation of legal rights also.

Q 98.   A occupied a government post and was superannuated last month. After his retirement, the post has been
lying vacant
and B, one of the clients of A had all his work stuck because of no signature from that office.
However, one day, B was
surprised to find C, the nephew of A, on that post, who lacked the basic qualification
for that post. Moreover, C refused to sign
B's paper, unless given Rs. 10,000 for the same. B approached
the Supreme Court, for this unjust action. Decide

a)  Supreme Court should issue the writ of habeas corpus to produce C in the court.

b)  Supreme Court should issue the writ of mandamus, asking C to perform his duties.

c)  Supreme Court should issue the writ of prohibition to stop C from eliciting bribe.

d)  Supreme Court should issue the writ of Quo Warranto, demanding justification for holding the post.

Q 99.   Which of the following option, states the correct difference between the writ of certiorari and writ of prohibition?

a)  Writ of prohibition is issued by the superior court and writ of certiorari is issued by Supreme Court only.
b)  
Writ of prohibition is issued to prevent the on-going judicial process, whereas the writ of certiorari is
issued to quash the
decision already taken.
c)  Writ of prohibition is to cure the wrong act already taken, whereas writ of certiorari is issued for preventive
measures.

d)  
Writ of prohibition is applicable to administrative authorities also, whereas writ of certiorari could be
issued only against judicia
bodies.

Q 100.   A and B entered a contract, wherein A had to deliver a consignment to B within a stipulated time. However, A
failed to
send it, due to some glitches in the manufacturing unit. B approached the Supreme Court, demanding
the writ of mandamus
to be issued against A. Decide.

a)  Writ of mandamus could be issued, demanding A to perform his promise.

b)  Writ of mandamus could be issued because A failed to perform his duties.

c)  Writ of mandamus could not be issued for the enforcement of a private right.

d)  Writ of mandamus could be issued only by the high court for the private right enforcement.

Directions for questions 66 to 105: You have been given some passages followed by questions based on each
passage.
You are required to choose the most appropriate option which follows from the passage.
Only the information given in the
passage should be used for choosing the answer and no external knowledge of
law howsoever prominent is to be applied.

Passage - 8

The aftermath of recent large-scale disasters in the 2020 back to back like the Australian Bushfire, Floods in
Indonesia, The
Dreaded Coronavirus, Volcano Eruption in the Philippines, Earthquakes in India, Turkey,
The Caribbean, China, Iran, Russia
Philippines, Locust Swarms In East Africa & Asia, Cyclone Amphan in India
& Bangladesh, Forest Fires in Uttarakhand, India
Floods In Assam in India, Snow in Antarctica Turns Green and
many such past incidents over a period of times realized the
importance of carefully planning for the unexpected
events and same goes when drafting the contracts. As upon happening of
such disastrous events it becomes
impossible for either party to perform its obligations under the contract then in such
scenario the Force Majeure
clause in the contract comes into picture. Force Majeure is a key tool in managing the risk of such
challenging
circumstances is the force majeure clause in the contract. A company must insert a force majeure clause into a
contract to relieve itself from liability in the event it cannot fulfil the terms of a contract for reasons which are
beyond its
control.

Force majeure clause aims at exempting a party from its obligations under a contract which has become impossible
for the
performance, due to intervention of a superior and unavoidable force. The concept of force majeure has
gained significance in
the past few years and even more now under the present COVID 19 pandemic the world is
going through everyone and every
business is effected and is forced to find new ways to run there business that
is why now force majeure clause in a contract
has gained much more importance then it had ever before.

The force majeure clause is a provision in a contract which allows both parties for non-performance of their
contractual duties
due to unavoidable circumstances and situation beyond human control thereby relieving the
parties from their respective
liabilities arising in the course of non-performance.

Though the above clause allows for rescue in chaotic cases like the present COVID 19 situation, its applicability
depends on
the specific use of words.

The essential constituents of force majeure clauses:


An unexpected/unforeseen intervening event occurred; Such an event has made the performance of the obligations
under the
contract impossible or impracticable; The parties have taken all measures to perform the obligations
under the agreement or
at least made sure that the damages are less;

The affected party claiming relief under force majeure, will have the burden of proof to show that the force majeure
event has
affected such party's performance of the contract.
Q 101.   State A is located in the south eastern part of Asia. The neighbours to its north B and C are in a state of war
owing to
the influx of illegal migrants and consequent occupation of territory. Merchant in state A, was bound
to supply the famous
apples of state A to a company in State Z, one of the eastern neighbours of State A,
however owing to the hostile condition in
the neighbourhood, merchant of state A refused to send the
consignment, invoking force majeure clause of their contract.
Decide the validity.

a)  Invoking clause force majeure is not valid because the hostility had not impacted their business.

b)  Invoking clause force majeure is not valid because state Z is not engaged in a war.

c)  Invoking clause of force majeure is valid because the war has impacted the apple cultivation.
d)  Invoking clause of force majeure is valid because all contracts come to an end in case of war.

Q 102.   A convention was to be held in State U on celebration of 15th year of diplomatic establishment. The convention
was
exclusively for the citizens of State P. The organisers have appointed Z who was supposed to book a
hall. From past few
weeks, neighbouring state R has been engaged in a military troop build up near the
border, raising concern. Out of the many
advisories that were issued by the government of state I, one read,
"It is unadvisable to travel to state U". However, many
citizens of state P went to the convention, but found
that Z has not booked any hall, the organisers faced heavy loss in paying
compensation to the people who
had travelled to State U. Z has claimed the defence of force majeure.

a)  Defence of force majeure cannot be taken, because the war has not started.
b)  Defence of force majeure cannot be taken because inadvisable is not same as impossible.

c)  Defence of force majeure can be taken because the state P has issued advisory against travel to U.

d)  Defence of force majeure can be taken because there was threat of war.

Q 103.   The government declared nationwide lockdown which has resulted in travel restriction for non-essential
purposes and
products, because of the rising cases of COVID-19. Meanwhile a dairy company,
which entered into a contract with retail store
to deliver certain quantity of milk, an essential item, during a
stipulated period, has refused to fulfil the contractual obligation
owing to the travel restrictions. Decide.

a)  Force majeure clause could be invoked because the situation is unforeseen and unprecedented.

b)  Force majeure clause could be invoked because the lockdown has resulted in travel restrictions.

c)  Force majeure clause could not be invoked because lockdown has not impacted the supply of essential
items.

d)  Force majeure clause could not be invoked because the consignment could have been delivered before
the lockdown.

Q 104.   Which of the following statement is not true with respect to the Force Majeure clause under the Indian
Contract Act?

a)  Force Majeure is covered under section 32 and 56 of the Indian Contract Act 1872.

b)  Force Majeure includes an act of God, natural disaster, war, labour unrest, epidemics and strikes.

c)  Parties of the contract, in case of force majeure must try to lessen the losses.

d)  Indian Contract Act 1872 does not provide for temporary suspension of performance of contract.

Q 105.   Which of the following is not an essential element of the force majeure?

a)  An unexpected/unforeseen intervening event is an essential element of force majeure.

b)  An event that has made the performance of contractual obligations impossible or impracticable.

c)  
The parties have taken all measures to perform the obligations under the agreement or at least made
sure that the damages
are less;
d)  The burden of proof to show force majeure event is on the defaulter party.

Logical Reasoning
Directions for questions 106 to 135: Read the passages and answer the questions that follow.

Passage - 1

Climate change impacts-among them, shifts in soil quality, precipitation, pest regimes, and seasonal growth
patterns, along
with land degradation and reduction in biodiversity-have impacted agricultural and aquatic food
production systems across the
world. Indeed, the causal links between climate change and food security are
manifesting more clearly, particularly in the
developing world, where nutrition deficiency is common, as is the
predominance of rain-fed, farm-centered agricultural
systems. This report reimagines the future of food, and how
countries of the Global South, including India, can build resilient
food systems. The issues of poor food safety, food
insecurity, and unsustainable food systems are closely interlinked. These
have been further exacerbated by climate
change and the Covid-19 pandemic, and have disproportionately affected some
populations more than others.
The importance of access to safe and nutritious food therefore cannot be overemphasized.
With a rapidly growing
human population and burgeoning demand for food, intensive use of agrochemicals-i.e. pesticides and
fertilizershas
become the norm to ramp up the production of livestock and crops. Without chemical pesticides and crop
protection in general, more than half of the world's crops would be destroyed by insects, weeds, and diseases.
Food
production per acre would decline rapidly; the area of land used for crop cultivation would have to increase.
This, in turn,
would have detrimental effects on wildlife habitats and ecosystems and dilute the quality of soil due to
erosion. There is also
the likelihood of a rise in food prices and reduced food output. It is pertinent to note that even
as developed countries use
80% of the pesticides produced globally, more than half of the recorded deaths due to
pesticides are reported in poorer
countries. This underscores the importance of food safety regulations; in the
Global South, there is a lack of occupational
safety standards on food composition, weak enforcement of rules
and regulations, and inadequate knowledge of pesticide
use. Therefore, the nature of the food being consumed is
taking a heavy toll on public health, more so in poor economies.
Food and agricultural regulations around the world
must pay attention not only to the increasing food demand and the need
for growth in production but also to the
safety of the food being produced.
Q 106.   According to the given passage, which of the following statements is False?

a)  
The issues of poor food safety, food, and unsustainable food systems were accelerated by Covid pandemic
and climate
change.

b)  Chemical contamination can lead to cancer

c)  The world requires to focus solely on fulfilling the demand of the population, hence howto increase the
production of food.
d)  Only 20 percent of the food produced by the developed country is free from pesticide

Q 107.   What is the central idea in the passage as conveyed by the author?

a)  Reimagining climate-resilient food systems.


b)  Intensive use of agrochemicals is a concern

c)  The pandemic has affected the demand for food in an underdeveloped country.

d)  Changes that are required in the Rules and regulation

Q 108.   It is pertinent to note that even as developed countries use 80% of the pesticides produced globally,
more than half of
the recorded deaths due to pesticides are reported in poorer countries. What does the
above statement convey?

a)  Only 20% of the pesticide is produced in the underdeveloped country

b)  Developed country witness no severe consequences of the use of pesticide

c)  Recorded death according to the report might be wrong.


d)  
It is very ironic to note that the maximum use of pesticide is by the developed country but still the one who
suffers more is the
countries that are poor.

Q 109.   "intensive use of agrochemicals-i.e. pesticides and fertilizers-has become the norm to ramp up production
of livestock
and crops". What isn't the cause for the increase in the use of agrochemicals?

a)  The rapid increase in the food demand

b)  Increase in the population

c)  crops would be destroyed by insects, weeds, and diseases otherwise

d)  Rules and regulations have not remained severe, hence the use of pesticides has been increasing
drastically.

Q 110.   What does the author trace about food security during the pandemic time?

a)  Non-privileged were effect during the covid time more than others
b)  Government should have emphasized the importance of access to the safe and nutritious food system

c)  There should have been an equal contribution by the developed and underdeveloped countries.

d)  The area of land used for crop cultivation would have to increase.

Directions for questions 106 to 135: Read the passages and answer the questions that follow.

Passage - 2

The latest list of 'unparliamentary' words published by the Lok Sabha secretariat has sparked sharp criticism from
Opposition
parties because it steps beyond the usual range of phrases that are frowned upon in Parliament and
also penalises the use of
language that is often used to target the government. While members of Parliament are
not banned from using these words,
as the Lok Sabha Speaker, Om Birla, has confirmed, the language will be
redacted from official transcripts. The list includes
terms like jumlajeevi (someone who repeatedly makes false
promises) and Snoopgate (a reference to allegations of
wiretapping), as well as everyday words and phrases
such as 'ashamed', 'corrupt', 'bloody', 'dictatorial' and 'sexual
harassment'. Two things are therefore clear.
First, the list of 'unparliamentary' words does not significantly reduce the
incentive for MPs to use those terms if
their audience comprises fellow legislators or ordinary Indians. Second, the purpose of
the list is not to keep
proceedings from turning rumbunctious but to eliminate records of contentious debates from being
referenced
publicly in the months and years ahead. That is where the problem lies. The British Parliament, too, has a list of
words deemed 'unparliamentary' - but these are limited to abuses like 'guttersnipe', 'rat' and 'hooligan'. In April, the
Speaker of
the House of Commons allowed MPs to call the then prime minister, Boris Johnson, a "liar". In the
United States of America, a
witness before the ongoing Congressional hearings into the January 6, 2021 riot at
Capitol Hill accused the former president,
Donald Trump, of using "lies, deceit and snake oil". There is a lesson in
these instances - for legislators to hold governments
accountable on behalf of the people, they must expose
wrongdoing. Punishing language needed for such accountability sits
naturally only in a dystopian world like the one
conjured by George Orwell in 1984, where the mode of communication,
Newspeak, is deliberately ambiguous and
has a limited vocabulary. Thought control - the aim of limiting language in Orwell's
novel - is not what any contemporary
legislature in genuine democracy practices. Challenging false promises and alleging
corruption is not
unparliamentary. Eliminating records of democratic debates is.
Q 111.   What is the central idea in the passage as conveyed by the author?

a)  The language used in the parliament should be edited before publishing.

b)  A parallel study of the unparliamentary words that are used in India and Britain.

c)  
The addition of fresh words underscores the need to reflect on the compatibility of the practice with
modern democratic ideals.

d)  
It is crucial for legislators to observe an acceptable level of conduct and eliminate unparliamentary words
as it disturbs
democracy.
Q 112.   "the list of 'unparliamentary' words does not significantly reduce the incentive for MPs to use those terms if
their
audience comprises fellow legislators or ordinary Indians." What is the effect of the rule "that the
language would be redacted
from the official transcript" on the member of parliament in India that was
implemented by the speaker Om Birla?

a)  It would not significantly motivate them to not use "unparliamentary" words.

b)  The purpose of the list is to stop unruly language in the parliament.

c)  It would cultivate harmony among fellow MP

d)  It affects the mindset of the Indian audience to choose significant representatives.

Q 113.   As per the passage, how America and Britain are managing to expunge the unparliamentary words?

a)  'guttersnipe', 'rat', and 'hooligan' are not unparliamentary words in Britain but abuses.

b)  ashamed', 'corrupt', 'bloody', 'dictatorial', and 'sexual harassment should be added to the British list too
as the list is limiting

c)  Punishing Donald Trump for using words such as "lies, deceit, and snake oil" makes the government
more accountable

d)  
The consequence of punishing Donald Trump for using words such as "lies, deceit, and snake oil"
makes the government
more powerful than the legislators

Q 114.   "Punishing language needed for such accountability sits naturally only in a dystopian world". What is the
relevant idea
beyond the given statement?

a)  The utopian world uses a language that sounds polite and facilitates the proper functioning of the government.

b)  In a utopian world, it is quite natural to expose the wrongdoings of a government as and when required.

c)  In a dystopic society, punishment is unnatural for using unparliamentary words.

d)  Dystopia facilitates the wrongdoing of the government through the restricted hold on the language.

Q 115.   â€œNewspeak, is deliberately ambiguous and has a limited vocabulary." what are the intentions of the author
while
considering George Orwell's novel to the Indian context?

a)  Broadcasting in the redacted language would limit democracy in India.

b)  Challenging the government would be difficult since the language becomes ambiguous and the truth
remains covered.

c)  Corruption would increase as there would be no allegations against the government


d)  All of the above.

Directions for questions 106 to 135: Read the passages and answer the questions that follow.

Passage - 3

The time may have come for a more concerted effort by the Government and the protesting farmers to find a
solution to the
prolonged impasse over the three agriculture-related laws enacted last year. The Supreme Court
has emphasised that public
roads cannot be blocked indefinitely by protesters. In an observation in the context of
more petitions from members of the
public, complaining that their right of free movement has been curtailed by the
ongoing protests by farmers, the Court has
said a solution has to be found, that roads cannot be blocked for long
and there is no reason for it to lay down the law again
and again. Even while denying that they are responsible for
the hardship faced by the public, and blaming the police for the
blockade, the farmers say they should be allowed
to continue their agitation at the Ramlila Maidan. The conflict between the
two competing rights is not the only
aspect that requires immediate attention. There seems to be no attempt to break the
deadlock on the core issue,
with the farmers demanding an outright repeal of the laws and statutory validation for the claim
that the MSP
regime will not come to an end. The two core demands have not seen any breakthrough. It is not out of place to
recall that an expert committee constituted by the Court has submitted its report, but nothing has been heard
about it after
that. To an extent, the fact that the Court has not taken it up again or made the report public may be
a factor in the continuing
impasse. An expedited hearing that involves further review of the panel's recommendations,
or any such similar initiative from
the Government, is needed to arrive at a solution. The objective must not be
merely to resolve the incidental issue of blocked
roads, but to reconcile or eliminate the deep differences over
what the Government sees as necessary reforms in the farm
sector. Any reform that seeks to eliminate distortions
in the sector must also win the confidence of farmers, the principal
stakeholders.
Q 116.   Which of the following action by the government will be MOST strongly endorsed by the author?

a)  Government must immediately accede to both the core demands of the farmers.

b)  Government is trying to sidestep the core issues and whiling time as the blockade continues.

c)  Government must enforce Supreme Court's orders by removing the blockade.

d)  Government needs to devise a solution for core issues, and not merely for road blocks.

Q 117.   What is the primary message of the author in the passage?

a)  Its' time to end the impasse. b)  The prolonged unnecessary impasse.

c)  The impasse over three farm laws. d)  The impasse overshadows key issues.

Q 118.   Which of the following can definitely not be inferred from the Supreme Court's observations about the
blockade?

a)  The reference was to last year's judgment on the Shaheen Bagh protest against the Citizenship
(Amendment) Act.
b)  Public ways and public spaces cannot be blocked indefinitely even during a peaceful protest.

c)  The observation can be seen as a disapproval of the tactic of holding interminable protests.

d)  
The observation can be seen as an implicit criticism of the Government for being unable to find a solution
to the farmers'
grievances.

Q 119.   Which two rights are being referred to in the statement "The conflict between the two competing rights is not
the only
aspect that requires immediate attention"?

a)  Freedom to select the protest venue and right to allocate the venue.

b)  Right to enact the legislation and freedom to protest unencumbered.

c)  Freedom to assemble peacefully and right to commute safely.

d)  Freedom to protest freely and the right to free movement of the public.

Q 120.   Which of the following if true would be an example of a goodwill gesture by the government towards the
farmers to
win their trust?

I. To not prosecute the farmers for the unsavory violent incidents caused by them on Republic Day when
the protest rolled into
the Capital

II. Compromise to a partial agreement on decriminalizing stubble-burning and safeguarding power subsidies
as demanded by
farmers.

a)  Only I b)  Only II c)  Both I and II d)  Neither I nor II

Directions for questions 106 to 135: Read the passages and answer the questions that follow.

Passage - 4

India began its journey as a cricketing nation at Lord's in 1932, but the after-effects of Partition were felt on the
cricket team
too as a few players who represented the country before Independence migrated to Pakistan.
Lala Amarnath became India's
first captain after Independence as the team embarked on its maiden tour of
Australia in November 1947. The opening pages
of Indian cricket history are a colourful, vibrant mosaic that
encompasses its strengths and weaknesses in equal measure. A
tale of individual triumphs in the backdrop of
religious identities, palace intrigues, divided loyalties, and colonial servitude
represented by a cast of characters
whose idiosyncrasies would be a delight to any fiction writer. The most popular and
revered face of the Pentangular,
CK Nayudu, was to become the first Indian Test captain, but not before palace intrigues and
vicious politics played
out in the full public display. Col Nayudu, as he is popularly known, was a big hitter of the ball whose
first scoring
shot in the Pentangular was a six. He was flamboyant and a brisk scorer of runs who had caught the public
imagination. It won't be wrong to say that he was the first superstar of Indian cricket. Yet, when the Indian team to
tour
England in 1932 was selected, he was not the captain. Fighting for that honour were two patrons of Indian
cricket, the
Maharaja of Patiala, Bhupinder Singh, and Maharaj Kumar of Vizianagram, better known as Vizzy.
Their merit for the job was
limited to their ability to open out their purse strings to fund the Indian team's travel
expenses. Vizzy lost out in the race and
the captain-select, Maharaja of Patiala, had to withdraw from the team
because of health issues. The captaincy fell on
Maharaja of Porbandar with Nayudu not even being his deputy.
A commoner was not thought fit to lead a national side, no
matter what his skills. However, better sense seemed
to prevail, and when the Test was played, both captain and vice-captain
withdrew, and Nayudu led an Indian side
that is better remembered for the wonderful debut of two of its fast bowlers,
Mohammed Nissar and Amar Singh.
Though India lost the Test, they played well enough not to embarrass themselves. Indian
cricket had arrived at the
international stage.
Q 121.   According to the given passage, which of the following statements is not true?

a)  The first match to be played by India was against Australia

b)  Few players were eliminated from the tour as an aftermath of the partition

c)  The beginning of the Indian cricket team was quite colorful and vibrant as it talks about strength.

d)  Lala Amarnath became India's first captain in cricket.

Q 122.   It was a human quest for excellence in the midst of intrigue and chaos for the players because of the
following
reasons:

a)  Religious difference b)  British rule c)   d)  All of the above


Interference of the royalties

Q 123.   According to the given passage, which of the following statements is true?

a)  Nayudu's merit was finally acknowledged.


b)  Maharaja of Porbandar became the captain because the commoners were not allowed.

c)  It would have been wrong to say that Col Nayudu was the first superstar of the Indian cricket team.

d)  Col Nayudu was not selected for the England tour.

Q 124.   What was not the backdrop of the Indian cricket team at the time of independence?

a)  A test was played, and both captain and vice-captain withdrew.

b)  Merit was limited to one who can fund the trips to play the matches.

c)  A commoner was not thought fit to lead a national side

d)  Col Nayudu was not the captain of the England tour

Q 125.   What context could be derived from the following paragraph?

a)  
It will help us understand better why and how cricket, intrigue, and corruption go hand-in-hand, and how
difficult it must have
been even back then to rise above personal agendas and political intrigues to forge
a cricket team purely on merit and
integrity.
b)  History of the Indian Cricket team

c)  Both (a) and (b)

d)  It helps us to understand the formation of the India Test team took place and Nayudu evolved.
Directions for questions 106 to 135: Read the passages and answer the questions that follow.

Passage - 5

The attempts to build socialism in some nations during the twentieth century have not been successful.
The promised land of
abundance, free from exploitative relationships, did not arrive. Instead, there was marked
tyranny through centralised control
of political power, the curbing of liberties, and the stagnation of individual
material prosperity (1). The critics of socialism
persistently maintain that there is no alternative to capitalism as
embodied by an economic system of private property,
markets, and individual incentives. Capitalism comes in
many variations on these themes, but it is claimed that whatever be
the variant, it is always preferred to socialism,
which is marked by excessive government control and restrictions on private
profits and property. The dark side of
socialist failures has been well-documented, publicised, and ingrained in public memory
over decades (2).

Two trends are, however, worth noting. Firstly, governments in societies marked by free markets and private
property are
becoming increasingly autocratic through controls over individual liberties and neutral institutions.
There is a clear rise in the
popularity of right- of-centre, illiberal regimes with strong fascist traits. Capitalist societies
are also showing an alarming
increase in inequalities of income and wealth. Poverty and unemployment have not
become things of the past. Uncertainties
of everyday living have risen sharply. The degradation of the natural
environment brought about by consumerism is leading
the planet towards a point of no return (3).

The second trend is, that despite its formidable critiques, the idea of socialism refuses to die. Many young people
are turning
away from consumerism as a sufficient condition for living well. For instance, many are not buying cars
or homes, are very
perturbed at the deterioration of nature, and believe that we are moving towards unacceptable
levels of inequality and
concentration of political power (4).

Is it possible to filter out a few enduring values that both capitalism and socialism have thrown up, such as liberty
and equality,
competition and cooperation? These could constitute the nucleus of thinking about the future where
all human beings could
thrive.

Even before that, the most important aspect of the good life has to lie in being in harmony with nature,
where human beings
as a species share the planet with other species. Without this transformation in our relationship
with nature, the world might
cease to be the way we have known it since the industrial revolution. Whether this can
be achieved without a major social and
political disruption remains a troubling question. Even if it were possible,
there would still remain questions of such a society's
dominant values and the role of its public institutions (5).
Q 126.   Another alternative for the term 'autocratic' as used in the passage is:

a)  septuagenarian b)  egalitarian c)  totalitarian d)  octogenarian

Q 127.   What is implied in statements 1 and 2?

a)  Socialism has failed in every instance according to the author

b)  Capitalism is the panacea of modern problems according to the author

c)  Socialism and Capitalism need to be synthesised

d)  Historically, socialism has failed to deliver in comparison to capitalism according to critics

Q 128.   The arguments made in statements 3 and 4 can be described as counter points to the assertions made
about:

a)  Capitalism b)  Socialism c)  Fascism d)  Consumerism

Q 129.   The assertions made in statement 5 are in complete agreement with claims made in which statement?

a)  1 b)  2 c)  1 and 2 d)  None of the above


Q 130.   In statement 4 the assertion being made is young people are not buying cars and homes. What social norm
are they
going against?

a)  Cars are considered essential b)  Owning a home provides security

c)  Acquisition of material wealth is healthy d)  Cars and homes are symbols of success

Directions for questions 106 to 135: Read the passages and answer the questions that follow.

Passage - 6

Perhaps the earliest known sources of the idea of a distinction between the southern and the northern regions of
the Indian
subcontinent lay in the scriptures and stories of Ramayana and Mahabharata (and possibly in the
Vedas). Sage Agastya in
both Ramayana and Mahabharata was said to have lived on the southern slopes of
Vindhya mountains which marked a
notional boundary between the north and the south. Subsequently, in texts
and oral traditions belonging to Buddhist, Jain, and
a number of Vedic philosophical traditions the distinction between
the South and the North persists and gets reiterated.
Certainly, in the last one millennium the Deccan which
territorially accounts for a major part of the south, has been
exceptionally successful in warding off repeated
attempts by the Northern plains to subsume its distinct socio-cultural
individuality. Its renowned prosperity during
medieval times attracted migrants in search of better opportunities, who
assimilated with the culture of the land to
a great extent. Some scholars argue that the newcomers were driven by an
instrumental reasoning that this
assimilation was necessary merely to gain legitimacy, authority and power. However, there is
much evidence to
suggest otherwise.

Initially, the eagerness among the newcomers to shed vestiges of their past identity and readily absorb the Deccani
culture
may have been prompted by the 1347 CE revolt of the Amiran-e Sadah, the Tughlaq era Grandees of the
Deccan. They were
driven by a compelling desire to dissociate culturally from the centre at Delhi in order to
fabricate an identity for themselves
and the 'nation' that they had created. An example of amalgamation with
existing practices can be seen in the architecture of
the Deccani sultanates in which the influences of south Indian
temple art and architecture are immediately apparent.
Q 131.   What can be seen as the main purpose behind writing this passage?

a)  To portray southern culture is superior to northern culture.


b)  To understand historical differences between the northern and southern.

c)  To understand how the Ramayana of the north is different from that of the south.

d)  None of them.

Q 132.   Which of the following is a valid assumption?

a)  The North has tried to subdue the cultural identity of the Deccani people.
b)  People migrated from north to south to escape a certain cultural hegemony.

c)  Both (a) and (b) are correct

d)  None of the above

Q 133.   Which of the following cannot be inferred from the passage?

a)  Agastya Muni resided in the Vindhyas.

b)  According to some scholars the people who shifted to the South wanted a different identity of their own.
c)  The cultural identity although was similar to the north, south always wanted to stay different.

d)  The monuments built by the Deccan sultanate had impressions of southern temples.

Q 134.   Read the following conclusions and select the correct option:

I. The mixture of sultanate inscriptions along with the influences of south Indian temple art is an example of
amalgamation.

II. Migrants from the north were not allowed to go back once they had crossed the Vindhyas.
a)  I is true b)  II is true c)  Both are true d)  Both are false

Q 135.   Which of the following is true about the cultural distinction between northern and southern India?

a)  The traditions of South and North are different from the beginning of time.

b)  The inscriptions in the early religious texts portray the traditional distinction between the two sections.

c)  Both (a) and (b)

d)  None of the above.

Mathematics
Directions for questions 136 to 140: Answer the questions on the basis of the information given below.

A builder initially employed 60 men to construct a cuboidal water tank of dimensions 20 m × 12 m × 8 m in a


housing
society in 35 days. The builder had enough daily wages for 60 men for 45 days when construction began.
The tank was fitted
with two inlet pipes - P1 and P2, and one outlet pipe P3. Pipe P1 and pipe P2 alone can fill the
tank in 32 minutes and 24
minutes respectively, and pipe P3 alone can empty the tank in 48 minutes.
Q 136.   If all the 60 men work together for 10 days and the builder has to complete the construction of the tank in
another 15
days, then how many additional men will be required after the first 10 days?

a)  100 b)  40 c)  60 d)  50

Q 137.   If all the 60 men work together for 10 days and the builder has to complete the construction of the tank in
another 15
days with additional men, then how long will the daily wages last for all the men after the first 10
days?

a)  31 days b)  23 days c)  19 days d)  21 days

Q 138.   If all the three pipes P1, P2 and P3 are opened simultaneously, then how much time will be taken to fill the
tank
completely?

a)  19 minutes 12 seconds b)  17 minutes 24 seconds c)  21 minutes 12 seconds d)  24 minutes 24 seconds

Q 139.   If the pipes P1 and P2 are opened for 8 minutes to fill the tank with water, then how much more quantity of
water can
be filled in the tank now?

a)  1120 cubic meter b)  740 cubic meter c)  800 cubic meter d)  960 cubic meter

Q 140.   All the three pipes P1, P2 and P3 are opened simultaneously and after 9 minutes inlet pipes P1 and P2 are
closed.
How much time will P3 take to empty the tank after the inlet pipes were closed?

a)  24 minutes 45 seconds b)  22 minutes 30 seconds c)  20 minutes 15 seconds d)  18 minutes 30 seconds

Directions for questions 141 to 145: Answer the questions on the basis of the information given below.

The table given below shows details regarding five investment schemes offered by a finance company. Vijay is an
investor
who has invested in each of these schemes.

Q 141.   Vijay invested Rs.5x in Scheme A and Rs.(6x - 2000) in Scheme B. If the interest earned from both schemes
are
equal, then find the total amount received from both schemes at maturity.

a)  Rs.1,12,000 b)  Rs.1,01,000 c)  Rs.1,23,000 d)  Rs.1,03,000

Q 142.   Vijay invested a certain amount in Scheme D for 4 years. The difference between the interest after 4 years
and at the
end of 2 years is Rs.5,082. Find the amount (in Rs,) invested by Vijay.

a)  24,000 b)  20,000 c)  16,000 d)  18,000

Q 143.   Vijay’s friend invested Rs.20,000 each in Scheme B and Scheme E, where interest is calculated half yearly.
What
is the difference between the interests earned from Schemes E and B at the end of one year?

a)  Rs.364 b)  Rs.196 c)  Rs.232 d)  Rs.224

Q 144.   If the interest earned from Schemes A and B are equal, then what is the difference between the principals
invested
by Vijay in Scheme C and Scheme A?

a)  Rs.30,000 b)  Rs.40,000 c)  Rs.35,000 d)  Rs.25,000

Q 145.   Vijay invested Rs.21,000 in Scheme D but withdrew one portion Rs.y at the end of the first year and the
remaining at
the end of the second year. If both the withdrawals are equal, then find the value of "y".

a)  Rs.12,300 b)  Rs.14,610 c)  Rs.13,310 d)  Rs.12,100

Directions for questions 146 to 150: Answer the questions on the basis of the information given below.

Even numbers among 1 to 9 are represented by letters a, b, c and d (not necessarily in this order), such that no
two letters
represent the same number.

Odd numbers among 1 to 9 are represented by p, q, r, s and t (not necessarily in this order), such that no two
letters represent
the same number.

Prime numbers among 1 to 9 are represented by l, m, n and o (not necessarily in this order), such that no two
letters
represent the same number.

Answer the following questions by solving these equations:


Equation 1: b × d + q = 49

Equation 2: a + c + o = 13

Equation 3: s × t + c = 25

Equation 4: l + m + p = 17

Equation 5: b × l + s = 47

Q 146.   What is the value of p?

a)  5 b)  7 c)  9 d)  3

Q 147.   

a)  x > 2

b)  

c)  x < 1

d)  
Q 148.   

a)  Greater than 9 b)  less than 8 c)  greater than 8 d)  less than 7

Q 149.   

a)  6 b)  8 c)  4 d)  10

Q 150.   What is the value of m?

a)  7 b)  2 c)  5 d)  3

You might also like